Download as pdf or txt
Download as pdf or txt
You are on page 1of 61

INSTA STATIC QUIZ

APRIL 2024

WWW.INSIGHTSONINDIA.COM INSIGHTSIAS
INSTA STATIC QUIZ
Table of Contents

1. POLITY ...................................................................................................................... 2

2. GEOGRAPHY ............................................................................................................. 10

3. ECONOMY................................................................................................................ 17

4. ART AND CULTURE ..................................................................................................... 24

5. HISTORY .................................................................................................................. 31

6. ENVIRONMENT.......................................................................................................... 37

www.insightsonindia.com 1
INSTA STATIC QUIZ

1. Polity
1) The notion of ‘Freedom’, in a society, implies
1. Absence of external constraints
2. Conditions in which people can develop their talents
3. All decisions are made collectively
How many of the above statements is/are incorrect?
a) Only one
b) Only two
c) All three
d) None

Solution: a)

Statement 3 is incorrect.

Both the aspects of freedom — the absence of external constraints as well as the existence of conditions in
which people can develop their talents — are important. A free society would be one which enables all its
members to develop their potential with the minimum of social constraints.

In order to be free, an individual should be able to make decisions individually, with a support of collective
decision-making in which no one individual dominates the others.

2) Consider the following statements regarding the word ‘sovereignty’ used in India’s Constitution.
1. The word sovereignty appears in the beginning of the Preamble to the Constitution of India.
2. Sovereignty is mentioned in the Constitution under Fundamental Duties.
3. It is also mentioned in the oaths for positions like the Chief Justices, Union Ministers and Members of
Parliament under the Third schedule of the Constitution.
How many of the above statements is/are correct?
a) Only one
b) Only two
c) All three
d) None

Solution: c)

The word sovereignty appears in the beginning of the Preamble to the Constitution of India, as the first attribute
of the independent republic of India. Its placement as the first among the core principles of the republic
underlines its importance in the Constitution.

Sovereignty is mentioned in the Constitution under Fundamental Duties, which are to be followed by all Indian
citizens but cannot be legally enforced. Article 51A(c) states that it is the duty of all citizens “to uphold and
protect the sovereignty, unity and integrity of India.”

It is also mentioned in the oaths for positions like the Chief Justices, Union Ministers and Members of
Parliament under the Third schedule: “…I will bear true faith and allegiance to the Constitution of India as by law
established, that I will uphold the sovereignty and integrity of India…”

3) Which of the following conditions precedent for the successful working of Democracy?
1. Equality of law and administration
2. Avoidance of tyranny of majority over minority
3. Presence of an opposition
4. Functioning of moral order in society
5. Public conscience
www.insightsonindia.com 2
INSTA STATIC QUIZ
How many of the above options are correct?
a) Only two
b) Only three
c) Only four
d) All five

Solution: d)

Conditions Precedent for the Successful Working of Democracy:


• Absence of glaring inequalities
• Presence of an opposition
• Equality of law and administration
• Observance of constitutional morality
• Avoidance of tyranny of majority over minority
• A functioning of moral order in society
• Public conscience

4) The Indian model of government is also called as the “Westminster‟ model of government because
a) The members of the parliamentary house are elected through adult franchise
b) Indian constitution was made with the assistance of the British
c) India follows parliamentary form of government
d) Indian constitution was modelled very closely on the lines of the British constitution

Solution: c)

Westminster is a place in London where the British Parliament is located. It is often used as a symbol of the British
Parliament.
The Parliament is the legislative organ of the Union government. It occupies a pre -eminent and central position in
the Indian democratic political system due to adoption of the parliamentary form of government, also known as
‘Westminster‘ model of government.

5) Consider the following statements regarding the Provisions in Indian Constitution.


1. The principle of Equality prohibits the State from discrimination against any citizen on grounds only of
religion, race, caste, sex and place of birth.
2. Constitution provides for reservation of posts in favour of any backward class of citizens, which in the
opinion of the State, is not adequately represented in the services under the State.
3. Constitution empowers the State to make any provision for the advancement of any socially and
educationally backward classes of citizens or for the Scheduled Castes and Scheduled Tribes.
How many of the above statements is/are incorrect?
a) Only one
b) Only two
c) All three
d) None

Solution: d)

Provisions of the Constitution:


• The relevant Constitutional provisions stand on two legs, which are mutually supportive. On the one hand,
there is the principle of Equality, which prohibits the State from discrimination against any citizen on
grounds only of religion, race, caste, sex, place of birth or any of them under Article 15(1), and guarantees
“equality of opportunity for all citizens in matters relating to employment or appointment to any office under
the State” under Article 16(1), in addition to prohibition against discrimination against any citizen on the
same grounds as in Article 15(1), specifically with respect to employment or appointment under the State.

www.insightsonindia.com 3
INSTA STATIC QUIZ
• The other leg is the special provisions, which under Article 15(4) empowers the State to “make any provision
for the advancement of any socially and educationally backward classes of citizens or for the Scheduled
Castes and Scheduled Tribes”, and under Article 16(4) provides “for the reservation of appointments or
posts in favour of any backward class of citizens, which in the opinion of the State, is not adequately
represented in the services under the State”.

6) The Indian constitution is a written one unlike in some of the other democracies. What does it imply?
1. All the laws made by Parliament are to be written down as a part of the constitution.
2. The form of government in India has been codified in the constitution to reduce political and
administrative conflicts.
3. Only because of a written constitution, citizens are able to enjoy fundamental rights.
How many of the above statements is/are correct?
a) Only one
b) Only two
c) All three
d) None

Solution: a)

Only Statement 2 is correct.

Constitution specifies the structure, organisation, powers and functions of both the Central and state
governments and prescribes the limits within which they must operate. Thus, it avoids the misunderstandings
and disagreements between the two. All the laws made in India are codified separate from the constitution and
maintained in a law book. They need not be a part of the constitution. Even in the UK where there is no written
constitution, people enjoy several fundamental rights. However, only because our fundamental rights are written
in the constitution, it is difficult to amend and change them as per the wishes of the political executive.

7) Which of the following from the Constitution of India is/are specially relevant to Social Justice and
Empowerment?
1. Preamble
2. First Schedule
3. Third Schedule
4. Fundamental Rights
5. Directive Principles of State Policy
How many of the above options is/are correct?
a) Only two
b) Only three
c) Only four
d) All five

Solution: b)

Options 1, 4 and 5 are correct.

Preamble: The words “JUSTICE, social, economic and political; EQUALITY of status and of opportunity; and to
promote among them all FRATERNITY assuring the dignity of the individual and the unity and integrity of the
Nation”.

First Schedule: It talks about Indian states and Indian territory.

Third Schedule: It contains forms of Oaths and affirmations for office holders.

Fundamental Rights:
A 23. Prohibition of traffic in human beings and forced labour
www.insightsonindia.com 4
INSTA STATIC QUIZ
A 24. Prohibition of employment of children in factories, etc.

Directive Principles of State Policy:


A38. State to secure a social order for the promotion of welfare of the people
A39 (only first part) - The State shall, in particular, direct its policy towards securing – (a) that the citizens, men
and women equally, have the right to an adequate means of livelihood

8) The Fundamental Rights further the idea of democracy because


1. They operate as limitations on the tyranny of the executive.
2. They secure vital political rights to the citizens of India.
Which of the above statements is/are incorrect?
a) 1 only
b) 2 only
c) Both 1 and 2
d) Neither 1 nor 2

Solution: d)

They operate as limitations on the tyranny of the executive implies that they limit the authority of the
government, and arbitrary laws of the legislature. They are justiciable in nature, that is, they are enforceable by
the courts for their violation.

Rights such as equality to contest for political office, right against discrimination etc show the political and social
equality of citizens.

9) The first amendment to the Constitution of India contained which of the following provisions?
1. Expanded the scope of restrictions on the right to free speech.
2. Provided protection for backward classes in the Constitution.
3. Exempted land reforms from scrutiny
How many of the above statements are correct?
a) Only one
b) Only two
c) All three
d) None

Solution: c)

The Constitution (First Amendment) Bill sought to make several consequential changes — from exempting land
reforms from scrutiny to providing protections for backward classes in the Constitution. Notably, it also
expanded on the scope of the restrictions on the right to free speech.

10) Consider the following statements.


1. Prices of milk in the country are decided by Cooperative and Private dairies based on cost of
production.
2. Improving animal husbandry and the quality of cattle breed is one of the Directive Principles of State
Policy in the Constitution of India.
3. ‘Preservation of cattle’ is a matter on which the Centre has exclusive powers to legislate.
How many of the above statements are correct?
a) Only one
b) Only two
c) All three
d) None

Solution: b)

www.insightsonindia.com 5
INSTA STATIC QUIZ

Statement 3 is incorrect.

Prices of milk in the country are decided by the Cooperative and Private dairies based on cost of production.
Under the distribution of legislatives powers between the Union of India and States under Article 246(3) of the
Constitution, the preservation of cattle is a matter on which the legislature of the States has exclusive powers
to legislate.

As per article 48 of Indian Constitution the state shall endeavour to organize agriculture and animal husbandry
on modern and scientific lines and shall in particular take steps for preserving improving the breed, and
prohibiting the slaughter, of cows and calves and other milch and draught cattle.

11) For filing Public Interest Litigation (PIL), the Supreme Court invokes which of the jurisdiction under them?
a) Advisory Jurisdiction
b) Writ Jurisdiction
c) Appellate jurisdiction
d) Original Jurisdiction

Solution: b)

In India, the PIL is a product of the judicial activism role of the Supreme Court. It was introduced in the early
1980s.
A Public Interest Litigation can be filed before the Supreme Court under Article 32 of the Constitution or before
the High Court of a State under Article 226 of the Constitution under their respective Writ Jurisdictions.

12) Consider the following statements regarding President’s rule that is imposed under Article 356 of the
Constitution.
1. It cannot be imposed without the written recommendation of the Governor of the concerned state.
2. Every proclamation of President’s rule must be approved by both the houses of Parliament within a
stipulated time.
Which of the above statements is/are incorrect?
a) 1 only
b) 2 only
c) Both 1 and 2
d) Neither 1 nor 2

Solution: a)

Article 356 empowers the President to issue a proclamation, if he is satisfied that a situation has arisen in which
the government of a state cannot be carried on in accordance with the provisions of the Constitution.
Notably, the president can act either on a report of the governor of the state or otherwise too (ie, even without
the governor’s report).

A proclamation imposing President’s Rule must be approved by both the Houses of Parliament within two months
from the date of its issue. If approved by both the Houses of Parliament, the President’s Rule continues for six
months.

13) Consider the following statements regarding Law Commission of India.


1. The Law Commission of India is a statutory body constituted by the Government of India from time to
time.
2. The commission is re-constituted every five years.
3. The Law Commission shall suo-motu, undertake research in law and review of existing laws in India.
Which of the above statements is/are incorrect?
a) 1, 2
b) 1, 3
www.insightsonindia.com 6
INSTA STATIC QUIZ
c) 2, 3
d) None of the above

Solution: a)

The Law Commission shall, on a reference made to it by the Central Government or suo-motu, undertake
research in law and review of existing laws in India for making reforms therein and enacting new legislations. It
shall also undertake studies and research for bringing reforms in the justice delivery systems for elimination of
delay in procedures, speedy disposal of cases, reduction in cost of litigation e tc.

The Law Commission of India is a non-statutory body constituted by the Government of India from time to time.
The Commission was originally constituted in 1955 and is re-constituted every three years.

14) Consider the following statements.


1. The Constitution gives the President of India the power to address either House or a joint sitting of the
two Houses of Parliament.
2. When the Constitution came into force, the President was required to address each session of
Parliament.
3. The 42nd Amendment of the Constitution made that the President shall address both Houses of
Parliament only once a year.
How many of the above statements is/are correct?
a) Only one
b) Only two
c) All three
d) None

Solution: b)

Statement 3 is incorrect.

The Constitution gives the President the power to address either House or a joint sitting of the two Houses of
Parliament.

Article 87 provides two special occasions on which the President addresses a joint sitting.
• The first is to address the opening session of a new legislature after a general election.
• The second is to address the first sitting of Parliament each year.

When the Constitution came into force, the President was required to address each session of Parliament. So,
during the provisional Parliament in 1950, President Prasad gave an address before every session. The First
Amendment to the Constitution in 1951 changed this position and made the President’s address once a year.

15) The term ‘fledgling democracy’ mean


a) Democracy without a written Constitution
b) A democracy that is not secular
c) An oldest democracy
d) A new Democracy

Solution: d)

The term fledgling is used to describe a person, organization, or system that is new or without experience.

16) Consider the following statements regarding Public Account of India.


1. Public Account of India accounts for flows for those transactions where the Union government acts as a
banker.
2. It does not include state provident funds and small savings deposits.
www.insightsonindia.com 7
INSTA STATIC QUIZ
3. Expenditures from Public Account of India require the approval of the parliament.
How many of the above statements is/are correct?
a) Only one
b) Only two
c) All three
d) None

Solution: a)

Only Statement 1 is correct.

Public Account of India accounts for flows for those transactions where the government is merely acting as a
banker. This fund was constituted under Article 266 (2) of the Constitution.

Examples of those are provident funds, small savings and so on. These funds do not belong to the government.
They have to be paid back at some time to their rightful owners. Because of this nature of the fund, expenditures
from it are not required to be approved by the Parliament.

17) Puttaswamy judgement is a landmark judgment of the Supreme Court of India that mainly deals with
a) Constitutionality of reservations
b) Uniform Civil Code
c) Right to privacy
d) Right to life

Solution: c)

Justice K. S. Puttaswamy (Retd.) and Anr. vs Union Of India And Ors is a landmark judgment of the Supreme Court
of India, which holds that the right to privacy is protected as a fundamental constitutional right under Articles 14,
19 and 21 of the Constitution of India.

18) Article 239A of the Constitution deals with


a) Power of President to make regulations for Union territories
b) High Courts for Union territories
c) Power of the Parliament to create a legislature for Union Territories.
d) Special provisions with respect to Delhi

Solution: c)

In 1962, after the signing of a treaty with France, the territories of Pondicherry, Karaikal, Mahe, and Yanam
became part of the Indian Union. In the same year, then Home Minister Lal Bahadur Shastri introduced a
Constitution Amendment Bill in Parliament, which grouped these territories into the Union Territory of
Pondicherry, and also inserted a new Article 239A into the Constitution. The new article gave Parliament power
to create a legislature for Union Territories.

19) Consider the following statements.


1. During the election period, the Election Commission of India (ECI) can order the government to observe
election related guidelines.
2. ECI can send Border Security Force (BSF) and Indo-Tibetan Border Police (ITBP) to states ahead of the
Assembly elections to maintain law and order during the election campaign.
3. When on election duty, government officers work under the control of the ECI and not the
government.
How many of the above statements is/are incorrect?
a) Only one
b) Only two
c) All three
www.insightsonindia.com 8
INSTA STATIC QUIZ
d) None

Solution: d)

• Very few election commissions in the world have such wide -ranging powers as the Election Commission of
India.
• EC takes decisions on every aspect of conduct and control of elections from the announcement of elections to
the declaration of results.
• It implements the Code of Conduct and punishes any candidate or party that violates it.
• During the election period, the EC can order the government to follow some guidelines, to prevent use and
misuse of governmental power to enhance its chances to win elections, or to transfer some government
officials.
• ECI can send Companies of central security forces to states ahead of the Assembly elections to maintain law
and order during the election campaign.

20) Consider the following statements.


1. Nominated members of the Rajya Sabha have the same rights and privileges as elected members of
Rajya Sabha with the right to vote in the election of the President.
2. The anti-defection law was not present in the original Constitution.
3. The Nominated members of the Rajya Sabha are subjected to disqualification, if they join a political
party within six months of being nominated to the House.
How many of the above statements is/are correct?
a) Only one
b) Only two
c) All three
d) None

Solution: a)

Only Statement 2 is correct.

In 1985 the Tenth Schedule, popularly known as the anti-defection law, was added to the Constitution.

The law specifies the circumstances under which changing of political parties by MPs invite action under the law.
The law covers three types of scenarios with respect to an MP switching parties.

The first is when a member elected on the ticket of a political party “voluntarily gives up” membership of such a
party or votes in the House contrary to the wishes of the party.

The second possibility is when an MP who has won his or her seat as an independent candidate after the election
joins a political party. In both these instances, the MP lose the seat in the House on changing (or joining) a party.

The third scenario relates to nominated MPs. In their case, the law specifies that within six months of being
nominated to the House, they can choose to join a political party.

www.insightsonindia.com 9
INSTA STATIC QUIZ

2. Geography
1) Consider the following statements.
1. Earth’s uppermost layer is the crust, which is the thinnest of all the layers.
2. Continental crust is less dense than ocean crust as the latter is made of basaltic rocks.
3. The innermost core of Earth is made only of molten iron.
How many of the above statements is/are correct?
a) Only one
b) Only two
c) All three
d) None

Solution: b)

Statement 3 is incorrect.

Just like an onion, Earth is made up of several concentric layers with one inside another. Earth’s uppermost layer
is the crust, which is the thinnest of all the layers. On the continental masses, it is about 35 km and on the ocean
floors only 5km.

Silica and alumina are the main minerals that constitute the continental mass, and are called sial (si, silica; al,
alumina). Silica and magnesium are the main minerals that constitute the oceanic crust, and are called sima (si,
silica; ma, magnesium).

Continental crust is less denser than ocean crust as the latter is made of basaltic rocks.
Core constitutes the innermost layer. Its radius is about 3500 km. Nickel and iron constitute the core, and it is
called nife (ni, nickel; fe, ferrous, i.e. iron). The temperature and pressure at the central core are very high. It is
the molten iron in core which gave rise to magnetic field of Earth.

2) Consider the following statements regarding Endogenic Forces.


1. Endogenic forces are those internal forces which derive their strength from the earth’s interior.
2. Endogenic forces are mainly land wearing forces.
3. The energy generated by radioactivity, rotational and tidal friction force emanating from within the
earth is the main force behind endogenic geomorphic processes.
How many of the above statements is/are correct?
a) Only one
b) Only two
c) All three
d) None

Solution: b)

Statement 2 is incorrect.

Endogenic Forces
• Endogenic forces are those internal forces which derive their strength from the earth’s interior and play a
crucial role in shaping the earth crust.
• Examples – mountain building forces, continent building forces, earthquakes, volcanism etc.
• The endogenic forces are mainly land building forces. The energy emanating from within the earth is the
main force behind endogenic geomorphic processes. This energy is mostly generated by radioactivity,
rotational and tidal friction and primordial heat from the origin of the earth.

Exogenic Forces

www.insightsonindia.com 10
INSTA STATIC QUIZ
• Exogenic forces are those forces which derive their strength from the earth’s exterior or are originated within
the earth’s atmosphere.
• Examples of forces – the wind, waves, water etc.
• Examples of exogenic processes – weathering, mass movement, erosion, deposition.
• Exogenic forces are mainly land wearing forces.

3) Consider the following statements regarding Paleomagnetism.


1. Paleomagnetism is the study of the record of the earth’s magnetic field with the help of magnetic fields
recorded in rocks, sediment, or archaeological materials.
2. Rocks formed from underwater volcanic activity are mainly basaltic which makes up most of the ocean
floor.
3. Paleomagnetic studies of rocks have demonstrated that the orientation of the earth’s magnetic field
has frequently alternated over geologic time.
How many of the above statements is/are correct?
a) Only one
b) Only two
c) All three
d) None

Solution: c)

• Paleomagnetism is the study of the record of the earth’s magnetic field with the help of magnetic fields
recorded in rocks, sediment, or archaeological materials.
• The polarity of the Earth’s magnetic field and magnetic field reversals are thus detectable by studying the
rocks of different ages.
• Rocks formed from underwater volcanic activity are mainly basaltic (low silica, iron-rich) which makes up
most of the ocean floor.
• Basalt contains magnetic minerals, and as the rock is solidifying, these minerals align themselves in the
direction of the magnetic field. This locks in a record of which way the magnetic field was positioned at the
time.
• Paleomagnetic studies of rocks have demonstrated that the orientation of the earth’s magnetic field
has frequently alternated (geomagnetic reversal) over geologic time. Paleomagnetism led the revival of the
continental drift hypothesis and its transformation into theories of Sea Floor Spreading and Plate Tectonics.

4) From the theory of continental drift, the forces behind the drifting of continents are:
1. Tidal force
2. Gravitational force
3. Pole-fleeing force
How many of the above statements is/are correct?
a) Only one
b) Only two
c) All three
d) None

Solution: c)

The first detailed theory of continental drift was proposed by Alfred Wegener in 1912. With the help of large
geologic and paleontological data, Wegener put forward that throughout most of geologic time period, there was
only one continent, that is called Pangea. In the Triassic Period, Pangea got fragmented, and the parts began to
separate and move from one another.
According to Wegener, the drift was in two directions:
1. Equatorwards due to the interaction of forces of gravity, pole-fleeing force (due to centrifugal force caused
by Earth’s rotation) and buoyancy, and

www.insightsonindia.com 11
INSTA STATIC QUIZ
2. Westwards due to tidal currents and this is because of Earth’s motion (According to Wegener, since Earth
rotates from west to east, the tidal currents act from east to west).

Wegener suggested that tidal force (gravitational pull of Moon and to a lesser extent, Sun) also played a major
role.
• The polar-fleeing force relates to the rotation of Earth. The shape of Earth is not a perfect sphere; there is a
bulge at the equator. The rotation of Earth causes this bulge.

5) Which of the following are block mountains?


1. Cascade Range in the USA
2. Vosges mountains in France
3. Great African Rift Valley
How many of the above statements is/are correct?
a) Only one
b) Only two
c) All three
d) None

Solution: b)

Statement 1 is incorrect.

Block Mountains
• Block mountains are created because of faulting on a large scale (when large areas or blocks of earth are
broken and displaced vertically or horizontally).
• The uplifted blocks are termed as horsts, and the lowered blocks are called graben.
• The Great African Rift Valley, The Rhine Valley and the Vosges mountain in Europe are examples.
• Block mountains are also called fault-block mountains since they are formed due to faulting as a result of
tensile and compressive forces.

6) Consider the following statements.


1. Lightning is a very rapid and massive discharge of electricity in the atmosphere, some of which is
directed towards the Earth’s surface.
2. It is generated in giant moisture-bearing clouds, whose base typically lies above 10 to 12 km from the
Earth’s surface.
Which of the above statements is/are incorrect?
a) 1 only
b) 2 only
c) Both 1 and 2
d) Neither 1 nor 2

Solution: b)

Lightning is a very rapid — and massive — discharge of electricity in the atmosphere, some of which is directed
towards the Earth’s surface. These discharges are generated in giant moisture-bearing clouds that are 10-12 km
tall. The base of these clouds typically lies within 1-2 km of the Earth’s surface, while their top is 12-13 km
away. Temperatures towards the top of these clouds are in the range of minus 35 to minus 45 degrees Celsius.

As water vapour moves upward in the cloud, the falling temperature causes it to condense. Heat is generated in
the process, which pushes the molecules of water further up.
As they move to temperatures below zero degrees Celsius, the water droplets change into small ice crystals. They
continue to move up, gathering mass — until they are so heavy that they start to fall to Earth.
This leads to a system in which, simultaneously, smaller ice crystals are moving up and bigger crystals are coming
down.

www.insightsonindia.com 12
INSTA STATIC QUIZ

Collisions follow, and trigger the release of electrons — a process that is very similar to the generation of sparks
of electricity. As the moving free electrons cause more collisions and more electrons, a chain reaction ensues.
This process results in a situation in which the top layer of the cloud gets positively charged, while the middle
layer is negatively charged. The electrical potential difference between the two layers is huge — of the order of a
billion to 10 billion volts. In very little time, a massive current, of the order of 100,000 to a million amperes, starts
to flow between the layers.
An enormous amount of heat is produced, and this leads to the heating of the air column between the two layers
of the cloud. This heat gives the air column a reddish appearance during lightning. As the heated air column
expands, it produces shock waves that result in thunder.

7) Consider the following statements regarding ocean temperature distribution.


1. Thermocline layer is characterised by rapid decrease in temperature with increasing depth.
2. In the Arctic and Antarctic circles, only one layer of cold water exists, which extends from surface to
deep ocean floor.
3. The oceans in the southern hemisphere record relatively higher temperature than in the northern
hemisphere.
How many of the above statements is/are correct?
a) Only one
b) Only two
c) All three
d) None

Solution: b)

Statement 3 is incorrect.

The thermocline layer lies below the first layer and is characterised by rapid decrease in temperature with
increasing depth.

In the Arctic and Antarctic circles, the surface water temperatures are close to 0° C and so the temperature
change with the depth is very slight. Here, only one layer of cold water exists, which extends from surface to
deep ocean floor.

The oceans in the northern hemisphere record relatively higher temperature than in the southern hemisphere .
The highest temperature is not recorded at the equator but slightly towards north of it.

8) Consider the following statements regarding Yarlung Tsangpo River.


1. It originates at Rongbuk Glacier in southern Tibet.
2. The River forms the world's deepest canyon, Yarlung Tsangpo Grand Canyon.
3. The River passes into the state of Arunachal Pradesh.
How many of the above statements is/are incorrect?
a) Only one
b) Only two
c) All three
d) None

Solution: a)

Statement 1 is incorrect.

The Yarlung Tsangpo is the upper stream of the Brahmaputra River located in the Tibet Autonomous
Region, China. It is the longest river of Tibet.

www.insightsonindia.com 13
INSTA STATIC QUIZ
Originating at Angsi Glacier in western Tibet, southeast of Mount Kailash and Lake Manasarovar, it later forms
the South Tibet Valley and Yarlung Tsangpo Grand Canyon before passing into the state of Arunachal
Pradesh, India.

Downstream from Arunachal Pradesh the river becomes far wider and is called the Siang. After reaching Assam,
the river is known as Brahmaputra. From Assam, the river enters Bangladesh.

When leaving the Tibetan Plateau, the River forms the world's largest and deepest canyon, Yarlung Tsangpo
Grand Canyon.

9) Which of the following events are most likely to happen when there is delay in the onset of monsoon over
Kerala?
1. The delay in the onset on monsoon leads to a poor monsoon or decrease in the amount of rainfall over
the Indian Subcontinent.
2. Delayed onset over Kerala invariably mean delay in the arrival of the monsoon over the entire country.
Select the correct answer code:
a) 1 only
b) 2 only
c) Both 1 and 2
d) Neither 1 nor 2

Solution: d)

The onset of the monsoon over Kerala marks the beginning of the four-month, June-September southwest
monsoon season over India, which brings more than 70 per cent of the country’s annual rainfall. The onset of the
monsoon is a significant day in India’s economic calendar.

Is it unusual for the monsoon to hit the Kerala coast early?


Neither early nor late onset of the monsoon is unusual, even though the forecast for this year is for earlier than
would be usually expected.

Does an early onset foretell a good monsoon?


No, it does not — just as a delay does not foretell a poor monsoon. The onset is just an event that happens
during the progress of the monsoon over the Indian subcontinent.
A delay of a few days, or perhaps the monsoon arriving a few days early, has no bearing on the quality or amount
of rainfall, or its regional distribution across the country, during the four-month monsoon season.

And does a delayed onset mean cascading delays across the country?
A delay in onset over Kerala can potentially delay the arrival of the monsoon in other parts of the country,
especially in the southern states, which normally start getting rain within days of the monsoon reaching the
Kerala coast.
But again, a delayed onset over Kerala does not automatically or invariably mean delays in the arrival of the
monsoon over the entire country.

10) Consider the following statements.


1. It is the ore which is used in manufacturing of aluminium.
2. It is mainly in tertiary deposits and is associated with laterite rocks.
3. It also occurs in the coastal tracts of India.
The above statements refer to
a) Nickel
b) Bauxite
c) Copper
d) None of the above

Solution: b)
www.insightsonindia.com 14
INSTA STATIC QUIZ

Bauxite is the ore which is used in manufacturing of aluminium. Bauxite is found mainly in tertiary deposits and is
associated with laterite rocks occurring extensively either on the plateau or hill ranges of peninsular India and also
in the coastal tracts of the country. Odisha happens to be the largest producer of Bauxite.

11) Consider the following statements.


1. Heat waves begin when high pressure in the atmosphere moves in and pushes warm air toward the
ground.
2. A hot spell is labelled as a ‘Heat Wave’ only when temperatures are above the historical average in an
area for more than one week.
3. When heat waves occur, the wind and cloud cover are minimised, which makes the air hotter.
How many of the above statements is/are incorrect?
a) Only one
b) Only two
c) All three
d) None

Solution: a)

Statement 2 is incorrect.

Temperatures must be above the historical average in an area for two or more days before the label “heat
wave” is applied to a hot spell. But the definition can vary by region;

Heat waves begin when high pressure in the atmosphere moves in and pushes warm air toward the ground.
That air warms up further as it is compressed, and we begin to feel a lot hotter.
The high-pressure system pressing down on the ground expands vertically, forcing other weather systems to
change course. It even minimizes wind and cloud cover, making the air more stifling. This is also why a heat
wave parks itself over an area for several days or longer.

12) Consider the following statements.


1. Wet-bulb temperature combines heat and humidity to indicate how much evaporation can be
absorbed into the air.
2. The wet bulb temperature is usually higher than the dry bulb temperature.
3. At wet-bulb temperatures above 35 degrees Celsius, it becomes difficult to reduce body temperature
via sweating.
How many of the above statements is/are correct?
a) Only one
b) Only two
c) All three
d) None

Solution: b)

Statement 2 is incorrect.

The wet bulb temperature is usually lower than the dry bulb temperature, and the difference between the two
increases dramatically as the air becomes dry.

The most worrying weather measurement is not the heat typically reported in forecasts but the wet-bulb
temperature, which combines heat and humidity to indicate how much evaporation can be absorbed into the
air. At wet-bulb temperatures above 35 degrees Celsius, we become unable to reduce our temperature via
sweating and will suffer potentially fatal heatstroke after only a few hours, even with shade and water.

www.insightsonindia.com 15
INSTA STATIC QUIZ
13) The factors that cause variations in insolation are:
1. The transparency of the atmosphere
2. The configuration of land in terms of its aspect
3. The length of the day
How many of the above statements is/are correct?
a) Only one
b) Only two
c) All three
d) None

Solution: c)

The energy received by the earth is known as incoming solar radiation which in short is termed as insolation.
The amount and the intensity of insolation vary during a day, in a season and in a year. The factors that cause
these variations in insolation are:
(i) The rotation of earth on its axis
(ii) The angle of inclination of the sun’s rays
(iii) The length of the day
(iv) The transparency of the atmosphere
(v) The configuration of land in terms of its aspect
Thus, all the given factors cause variation in the amount of insolation.

14) The twinkling of a star is due to


a) Change in the position of the star
b) Atmospheric reflection of starlight
c) Atmospheric refraction of starlight
d) Oscillation of starlight

Solution: c)

The twinkling of a star is due to atmospheric refraction of starlight. The starlight, on entering the earth’s
atmosphere, undergoes refraction continuously before it reaches the earth.

15) Consider the following statements regarding Tornadoes.


1. Tornadoes are whirling, vertical air columns that form from thunderstorms, without stretching to the
ground.
2. Changes in the wind’s speed and direction can lead to strong tornadoes.
Which of the above statements is/are incorrect?
a) 1 only
b) 2 only
c) Both 1 and 2

www.insightsonindia.com 16
INSTA STATIC QUIZ
d) Neither 1 nor 2

Solution: a)

Tornadoes are whirling, vertical air columns that form from thunderstorms and stretch to the ground. They
travel with ferocious speed and lay waste to everything in their path.

When winds vary in speed or direction at different altitudes–a condition known as wind shear–the updraft will
start to spin.
These changes in winds produce the spin necessary for a tornado. For especially strong tornadoes, changes are
needed in both the wind’s speed and direction.

3. Economy
1) Which of the following is/are the major traits of Recovery phase business cycle?
1. An upturn in aggregate (total) demand.
2. Inflation moves upward making borrowing cheaper for investors.
3. Unemployment rate starts increasing.
How many of the above statements is/are correct?
a) Only one
b) Only two
c) All three
d) None

Solution: b)

Statement 3 is incorrect.

Recovery
An economy tries to come out of the low production phase to survive. The low production phase might be
depression, recession or slowdown with the former being the worst and rare, governments take many new fiscal
and monetary measures to boost demand and production and ultimately a recovery in an economy is managed.
The business cycle of recovery may show the following major economy traits:
(i) an upturn in aggregate (total) demand which has to be accompanied by increase in the level of production;
(ii) production process expands and new investments become attractive;
(iii) as demand goes upward, inflation also moves upward making borrowing cheaper for investors;
(iv) with an upturn in production, new employment avenues are created and unemployment rate starts
declining; etc.

2) Consider the following statements regarding Federal Funds Rate (FFR).


1. The FFR is the interest rate at which commercial banks in the US borrow from each other overnight.
2. Like in India, where the RBI decides the repo rate, the Federal Reserve directly specifies the Federal
Funds Rate.
3. When the Federal Reserve wants to raise the prevailing interest rates in the US economy, it reduces the
money supply.
How many of the above options are correct?
a) Only one
b) Only two
c) All three
d) None

Solution: b)
www.insightsonindia.com 17
INSTA STATIC QUIZ

Statement 2 is incorrect.

What is the Federal Funds Rate (FFR) and how does the US Fed tweak it?
The FFR is the interest rate at which commercial banks in the US borrow from each other overnight. Arguably, if
banks borrow at higher rates from each other, they will also lend the money to consumers at a higher rate.
Now, unlike in India, where the RBI decides what the repo rate (or the interest rate at which RBI lends money to
banks) will be, in the US, the Fed can’t directly specify the FFR. Instead, it tries to “target” the FFR by controlling
the money supply. So when the Fed wants to raise the prevailing interest rates in the US economy, it reduces
the money supply. This forces every bank in the economy to charge higher interest rates. The process starts with
commercial banks charging higher interest rates to lend to each other for overnight loans.

3) Which of the following explains the U-shaped economic recovery?


a) The growth rises sharply from the lows much higher than the trend-line and stays there.
b) It is the scenario in which growth after falling, stagnates at low levels and does not recover for a long,
long time.
c) It is a scenario in which the economy, after falling, struggles and muddles around a low growth rate for
some time, before rising gradually to usual levels.
d) The economy which quickly rises like a phoenix after a crash.

Solution: c)

The Z-shaped recovery is the most-optimistic scenario in which the economy quickly rises like a phoenix after a
crash. It more than makes up for lost ground (think revenge -buying after the lockdowns are lifted) before settling
back to the normal trend-line, thus forming a Z-shaped chart.

V-shaped recovery in which the economy quickly recoups lost ground and gets back to the normal growth trend -
line.

A U-shaped recovery — resembling a bathtub — is a scenario in which the economy, after falling, struggles and
muddles around a low growth rate for some time, before rising gradually to usual levels.

A W-shaped recovery is a dangerous creature — growth falls and rises, but falls again before recovering yet again,
thus forming a W-like chart.

The L-shaped recovery is the worst-case scenario, in which growth after falling, stagnates at low levels and does
not recover for a long, long time.
Then, there is the J-shaped recovery, a somewhat unrealistic scenario, in which growth rises sharply from the
lows much higher than the trend-line and stays there.

4) Consider the following statements regarding Factor Income.


1. Factor income is income received from the factors of production.
2. It can be used to expose disparities in income distribution.
3. Factor income and current transfers together are referred to as the national income.
How many of the above statements is/are correct?
a) Only one
b) Only two
c) All three
d) None

Solution: b)

Statement 3 is incorrect.

www.insightsonindia.com 18
INSTA STATIC QUIZ
Factor income is the flow of income that is derived from the factors of production—the general inputs required
to produce goods and services.
Factor income on the use of land is called rent, income generated from labor is called wages, and income
generated from capital is called profit.
The factor income of all normal residents of a country is referred to as the national income , while factor income
and current transfers together are referred to as private income.
Factor income is most commonly used in macroeconomic analysis, helping governments to determine the
difference between gross domestic product (GDP) and gross national product (GNP).
It can also be used to expose disparities in income distribution.

5) ‘Money illusion’, a phrase coined by Keynes, is mainly caused by an ignorance of the real detrimental effect of
a) recession
b) inflation
c) low interest rates
d) high unemployment

Solution: b)

Money illusion is an economic theory positing that people have a tendency to view
their wealth and income in nominal dollar terms, rather than in real terms. In other words, it is assumed that
people do not take into account the level of inflation in an economy, wrongly believing that a dollar is worth
the same as it was the prior year.
Money illusion is sometimes also referred to as price illusion.

6) Consider the following statements.


1. India’s services sector output is measured by the S&P Global India Services Purchasing Managers’
Index (PMI).
2. Services Purchasing Managers’ Index (PMI) also provides the stock market an indicator of whether the
current economy is healthy or not.
3. The Reserve Bank of India does not look at Purchasing Managers’ Index (PMI) data to make interest
rate decisions.
How many of the above statements are correct?
a) Only one
b) Only two
c) All three
d) None

Solution: b)

Statement 3 is incorrect.

India’s services sector output, as measured by the S&P Global India Services Purchasing Managers’ Index
(PMI), rebounded from a three-month low in June to record a 13-year high of 62.3 in July.

The Services PMI provides advanced insight into the services sector, giving investors a better understanding of
business conditions and valuable information about the economic backdrop of various markets.

Services PMI also provides the stock market an indicator of whether the current economy is healthy or not,
which generally translates to higher corporate profits.

The PMI data are also used by the Reserve Bank of India to help make interest rate decisions.

7) The actual liabilities of the Union government include which of the following
1. Borrowings by PSUs
2. Loans taken for the recapitalisation of banks
www.insightsonindia.com 19
INSTA STATIC QUIZ
3. Capital expenditures of various Ministries.
How many of the above statements is/are correct?
a) Only one
b) Only two
c) All three
d) None

Solution: b)

Statement 3 is incorrect.

In addition to the borrowings by PSUs, the actual liabilities of the government would include loans taken for the
recapitalisation of banks.

Capital expenditure creates assets for the government and causes reduction in liabilities for the government.

8) Consider the following statements.


1. Disinvestment involves Selling minority shares of Public Enterprises, to public or private entity.
2. When the government sells majority shares in a public sector entity, that is strategic disinvestment.
3. Under strategic disinvestment, the government transfers the ownership and control of a public sector
entity to another public entity and the private sector is not involved here.
How many of the above statements is/are correct?
a) Only one
b) Only two
c) All three
d) None

Solution: b)

Statement 3 is incorrect.

What is strategic disinvestment or strategic sale?


When the government decides to transfer the ownership and control of a public sector entity to some other
entity, either private or public, the process is called strategic disinvestment.

What is the difference between strategic disinvestment/sale and disinvestment


Selling minority shares of Public Enterprises, to another entity be it public or private is disinvestment. In this
the government retains ownership of the enterprise. On the other hand, when the government sells majority
shares in an enterprise, that is strategic disinvestment/sale. Here, the government gives up the ownership of the
entity as well.

9) Consider the following statements regarding zero-coupon bond.


1. A zero-coupon bond is a debt security that not pay interest but instead trades at a deep discount,
rendering a profit at maturity, when the bond is redeemed for its full-face value.
2. These are special types of bonds issued only by the Central government specifically to a particular
institution.
Which of the above statements is/are incorrect?
a) 1 only
b) 2 only
c) Both 1 and 2
d) Neither 1 nor 2

Solution: b)

www.insightsonindia.com 20
INSTA STATIC QUIZ
A zero-coupon bond is a debt security that does not pay interest but instead trades at a deep discount, rendering
a profit at maturity, when the bond is redeemed for its full-face value.

It does not make periodic interest payments or have so-called coupons, hence the term zero coupon bond. When
the bond reaches maturity, its investor receives its par (or face) value.

Zero coupon bonds by private companies are normally issued at discount.

10) Which of the following can occur in economy of India due to deficit financing by the government?
1. Increased private investments
2. Inflation
3. Increase in money supply
4. Rise in employment rates
How many of the above options is/are correct?
a) Only one
b) Only two
c) Only three
d) All four

Solution: d)

The term ‘deficit financing’ is used to denote the direct addition to gross national expenditure through budget
deficits, whether the deficits are on revenue or on capital account.
Deficit financing in India is said to occur when the Union Government’s current budget deficit is covered by the
withdrawal of cash balances of the government and by borrowing money from the Reserve Bank of India.
Thus, in both cases, ‘new money’ comes into circulation. It is to be remembered here that government borrowing
from the public by selling bonds is not to be considered as deficit financing.

It is said that deficit financing is inherently inflationary. Since deficit financing raises aggregate expenditure and,
hence, increases aggregate demand, the danger of inflation looms large.

During inflation, private investors go on investing more and more with the hope of earning additional profits.
Seeing more profits, producers would be encouraged to reinvest their savings and accumulated profits as well
as increases the employment rate. Such investment leads to an increase in income—thereby setting the process
of economic development rolling.

11) Consider the following statements.


1. During colonial times, the Indian rupee was virtually linked to gold rather than British pound to which it
earned through exports.
2. The Reserve Bank of India (RBI) rules does not allow cross border trade transactions in rupee.
Which of the above statements is/are incorrect?
a) 1 only
b) 2 only
c) Both 1 and 2
d) Neither 1 nor 2

Solution: a)

As for India, currency hierarchy goes back to colonial times when the Indian rupee was virtually linked to the
British pound rather than to gold which it earned through exports.

Reserve Bank of India (RBI) allows cross border trade transactions in rupee which is a step towards
internationalisation of the currency.

12) The Government carries out Disinvestment to achieve which of the following goals?
www.insightsonindia.com 21
INSTA STATIC QUIZ
1. Introduce competition with market discipline
2. Reduce Financial burden on Government
3. Encourage open market to have wide share in ownership
4. Depoliticise essential services
How many of the above statements is/are correct?
a) Only one
b) Only two
c) Only three
d) All four

Solution: d)

13) Consider the following statements regarding Gold ETF.


1. Gold ETFs are passive investment instruments that are based on gold prices and invest in gold bullion.
2. The price of One Gold ETF unit is equal to 1 gram of gold of at least 99.5 percent purity.
3. Gold ETFs can be redeemed for physical gold.
How many of the above statements is/are incorrect?
a) Only one
b) Only two
c) All three
d) None

Solution: a)

Statement 3 is incorrect.

A Gold ETF is an exchange-traded fund (ETF) that aims to track the domestic physical gold price. They are passive
investment instruments that are based on gold prices and invest in gold bullion.

One Gold ETF unit is equal to 1 gram of gold and is backed by physical gold of very high purity. Gold ETFs
combine the flexibility of stock investment and the simplicity of gold investments.

Gold ETFs are listed and traded on the National Stock Exchange of India (NSE) and Bombay Stock Exchange Ltd.
(BSE) like a stock of any company. Gold ETFs trade on the cash segment of BSE & NSE, like any other company
stock, and can be bought and sold continuously at market prices.

www.insightsonindia.com 22
INSTA STATIC QUIZ
Buying Gold ETFs means you are purchasing gold in an electronic form. You can buy and sell gold ETFs just as you
would trade in stocks. When you actually redeem Gold ETF, you don’t get physical gold, but receive the cash
equivalent. Trading of gold ETFs takes place through a dematerialised account (Demat) and a broker, which
makes it an extremely convenient way of electronically investing in gold.
Because of its direct gold pricing, there is a complete transparency on the holdings of a Gold ETF. Further due to
its unique structure and creation mechanism, the ETFs have much lower expenses as compared to physical gold
investments.

14) Which of the following can be considered as ‘black swan events’ based on the terminology popularised by the
author Nassim Nicholas Taleb?
1. Fall of the Soviet Union
2. 2008 global financial crisis
3. Terrorist attack in the US on September 11, 2001
How many of the above statements is/are correct?
a) Only one
b) Only two
c) All three
d) None

Solution: c)

A black swan is a rare, unpredictable event that comes as a surprise and has a significant impact on society or
the world. These events are said to have three distinguishing characteristics – they are extremely rare and
outside the realm of regular expectations; they have a severe impact after they hit; and they seem probable in
hindsight when plausible explanations appear.

The black swan theory was put forward by author and investor Nassim Nicholas Taleb in 2001, and later
popularised in his 2007 book – The Black Swan: The Impact of the Highly Improbable.

When have such events occurred in the past?


Interestingly, Taleb’s book predated the 2008 global financial crisis – a black swan event triggered by a sudden
crash in the booming housing market in the US. The fall of the Soviet Union, the terrorist attack in the US on
September 11, 2001, also fall in the same category.

Is the Covid-19 pandemic a black swan event?


Taleb does not agree with those who believe it to be one. In an interview he called it a “white swan”, arguing that
it was predictable, and there was no excuse for companies and governments not to be prepared for something
like this.

15) The balance of payments includes both the current account and capital account. Consider the following
statements regarding this
1. The capital account is included in calculations of national output, while the current account is not.
2. The current account includes a nation’s net trade in goods and services, its net earnings on cross -
border investments, and not its net transfer payments.
Which of the above statements is/are correct?
a) 1 only
b) 2 only
c) Both 1 and 2
d) Neither 1 nor 2

Solution: d)

The balance of payments divides transactions into two accounts: the current account and the capital
account. The current account includes transactions in goods, services, investment income, and current
transfers.
www.insightsonindia.com 23
INSTA STATIC QUIZ

The capital account, broadly defined, includes transactions in financial instruments and central bank reserves.

The current account is included in calculations of national output, while the capital account is not.

4. Art and Culture


1) Consider the following statements regarding prehistoric Dancing Girl sculpture.
1. The Dancing Girl figurine was discovered in Mohenjodaro after India’s independence.
2. Even though Mohenjodaro and Harappa became part of Pakistani territory after the Partition, the
Dancing Girl remained in India as part of an agreement.
3. The sculpture was made in lost-wax casting method.
How many of the above statements are correct?
a) Only one
b) Only two
c) All three
d) None

Solution: b)

Statement 1 is incorrect.

The Dancing Girl figurine was discovered in Mohenjodaro in 1926.

Even though Mohenjodaro and Harappa became part of Pakistani territory after the Partition, the Dancing Girl
remained in India as part of an agreement. Today, the bronze figurine sits in the National Museum of India.

The Dancing Girl is evidence of the civilisation’s knowledge of metal blending and lost-wax casting – a
complicated process by which a duplicate sculpture is cast from an original sculpture to create highly detailed
metallic artefacts.

2) Which of the following are prominent features of the Nagara Architecture, in contrast to other types of Hindu
temple architecture?
1. Presence of large Gopurams
2. Absence of Mandapa
3. Shikhara above Sanctum
How many of the above statements is/are correct?
a) Only one
b) Only two
c) All three
d) None

Solution: a)

Only Statement 3 is correct.

This table gives a brief comparison between Nagara and Dravidian architecture.

www.insightsonindia.com 24
INSTA STATIC QUIZ

3) Hoysala monuments are found in


a) Hampi and Hospet
b) Sringeri and Dharwar
c) Mysore and Bangalore
d) Halebid and Belur

Solution: d)

Hoysala temples are found in Halebid and Belur in Karnataka. Halebid was the capital of Hoysalas at the later
stage, initially being Belur.

4) Consider the following statements regarding the paintings found at Ajanta Caves?
1. The themes of the paintings are the events from the life of the Buddha.
2. Figures in these caves are painted with over-stylisation, with less importance to naturalism.
3. The other important paintings are the famous Padmapani and Vajrapani.
How many of the above statements are correct?
a) Only one
b) Only two
c) All three
d) None

Solution: b)

Statement 2 is incorrect.

The themes of the paintings are the events from the life of the Buddha, the Jatakas and the Avadanas. Some
paintings such as Simhala Avadana, Mahajanaka Jataka and Vidhurpundita Jataka cover the entire wall of the
cave.
The other important paintings are the famous Padmapani and Vajrapani. Figures in these caves are painted
with considerable naturalism and there is no over-stylisation.

5) Consider the following statements regarding the classical dance ‘Mohiniyattam’.


1. Mohiniyattam is essentially a group dance performance by women.
www.insightsonindia.com 25
INSTA STATIC QUIZ
2. The Lasya aspect of dance is dominant.
3. It is based on secular themes.
How many of the above statements is/are correct?
a) Only one
b) Only two
c) All three
d) None

Solution: a)

Only Statement 2 is correct.

Mohiniyattam is essentially a solo dance performance by women in the state of Kerala. Mohiniyattam combines
the grace and elegance of Bharatanatyam with the vigour of Kathakali. There is a marked absence of thumping of
footsteps and the footwork is gentle.
Mohiniyattam generally narrates the story of the feminine dance of Vishnu. The Lasya aspect (beauty, grace) of
dance is dominant in a Mohiniyattam recital. Hence, it is mainly performed by female dancers.

6) What are the differences between the Hindustani and Carnatic styles of classical music?
1. Hindustani music uses the concepts of Sruti and Swara which are absent in Carnatic music.
2. Origin of Carnatic music can be traced to Vedas; whereas Hindustani music is Medieval in origin.
Which of the above statements is/are incorrect?
a) 1 only
b) 2 only
c) Both 1 and 2
d) None of the above

Solution: c)

These are two sub-genres of Indian music.


Similarities between both
• Similar concepts like sruti, raga, swara, taal
• Origin of both can be traced to Vedas (SamaVeda) - The oldest hindu scriptures - and to 13-14th century.
• Both monophonic in nature i.e. based around a single melody line, quite elaborate and expressive
However, differences exist in nature, style of singing, instruments and technique involved.
Differences
Hindustani Carnatic
• North India • South India

• Instruments uses are sitar, tabla, • Instruments used are violin, veena,
sarangi, Shehnai etc. gottu, mridangam
• Chief source is Sangit Ratnakara of • Chief source is saint Puranasara Das
Sarangdeva and carnatic trinity
• Emphasis on vocal music: songs are
composed to be sung

• Number of ragas less • Meant to be performed on singing


• It is not kirti based, it is raga based style when played on instruments
• Separate repertoire for instrumental • Number of ragas is more - 72
and vocal ragakarta
• It has Persian and Islamic influence • It is kirti based while Hindustani
and is a cultural synthesis of vedic raga based
chanting and folk music

www.insightsonindia.com 26
INSTA STATIC QUIZ
• Less rhythmic and less structured • More rhthmically structured and
• Accompanist do not play a major role intensive than Hindustani
• It can be sung in various styles known • Accompanists have much larger role
as gharanas such as gwalior gharana, • It is sung in a single style
jodhpur gharana etc.

7) Consider the following pairs


List I List II
1. Brihaddeshi Sarangadeva
2. Sangeeta Makaranda Narada
3. Swaramela-Kalanidhi Ramamatya
Which of the pairs given above is/are matched correctly?
a) 1 and 2 only
b) 2 and 3 only
c) 3 only
d) None of the above

Solution: b)

Brihaddeshi written in the 9th century by Matanga focused on the definition of the word ‘raga’.
The 11th century text, Sangeeta Makaranda was composed by Narada (Nanda) who enumerated 93 ragas and
classified them into feminine and masculine forms.
Swaramela-Kalanidhi written by Ramamatya in the 16th century which again deals primarily with ragas.

8) Which of the following is/are common between Dhrupad and Khyal genre of Indian classical music?
1. Both come from the court of Amir Khusrou of the 13th century.
2. Both are associated with the extensive use of the Been and Pakhawaj musical instruments.
Select the correct answer using the codes below.
a) 1 only
b) 2 only
c) Both 1 and 2
d) None of the above

Solution: d)

Both statements are wrong. The description of the two styles below will clarify the difference.
“ Dhrupad is believed to have been a further elaboration of the prabandha structure. While it might have had an
impetus for popularity even by the 14th century, it finds a blossoming period from 15th century onwards to about
the 18th century. During these centuries we meet the most respected and renowned singers and patrons of this
form. There was Man Singh Tomar, the Maharaja of Gwalior. It was he who was mainly responsible for the
enormous vogue of dhrupad.
The dhrupad is even now highly respected and can be heard on the concert platform but more often in temples of
North India. The dhrupad has somewhat receded to the background and is not so popular with the masses. The
Been and Pakhawaj which were closely associated with the dhrupad also do not find much patronage these days.
Today the pride of place in classical Hindustani Music is occupied by the Khyal. We are really not sure about the
beginning of the Khyal. The word is alien and means 'imagination'. And as you will find when you hear it is more
lyrical than the dhrupad. But whether the musical form itself is foreign. is a matter of doubt.
Some scholars are of the opinion that in fact, it has its roots in the ancient Indian roopaka alaps. It is also said that
Amir Khusrou of the 13th century gave it an impetus. Sultan Mohammed Sharkhi of the 15th century is credited
with encouraging this form. However, it attained its maturity at the hands of Niyamat Khan Sadarang and Adarang
of the 18th century.”

9) With reference to the semi-classical music form Thumri, consider the following statements
1. It is inspired by Bhakti Movement
www.insightsonindia.com 27
INSTA STATIC QUIZ
2. It is based on the romantic literature
3. Thumri is one of the division of Carnatic music
How many of the above statements is/are correct?
a) Only one
b) Only two
c) All three
d) None

Solution: b)

Statement 3 is incorrect.

Indian Hindustani Classical Music has various divisions like Khyaal, Thumari/Thumri, Tappa, Tarana, etc. The
thumri is based on the romantic-devotional literature inspired by the bhakti movement.

The text is usually derived from the Radha-Krishna theme and is of primary importance. The semi-classical music
form ‘thumr is known to have derived its name from thumakna. Loosely translated the word means, “dance -like
movements”.

Thumri was about mild eroticism and dramatic gestures and was the invention of courtesans. The text is romantic
and devotional in nature, and usually revolves around a girl’s love for Krishna. The language is a dialect of Hindi
called Brij bhasha.

10) Which of the following personalities is/are famous for the Hindustani Music?
1. Bhimsen Joshi
2. Gangubhai Hangal
3. M. S. Subbalakshmi
How many of the above statements is/are correct?
a) Only one
b) Only two
c) All three
d) None

Solution: b)

Statement 3 is incorrect.

Pandit Bhimsen Gururaj Joshi was a legendary Indian vocalist from Karnataka in the Hindustani classical tradition.
He is known for the khayal form of singing, as well as for his popular renditions of devotional music. He received
the Bharat Ratna, India's highest civilian honour, in 2009.

Gangubai Hangal was an Indian singer of the khyal genre of Hindustani classical music, who was known for her
deep and powerful voice. Hangal belonged to the Kirana gharana.

Madurai Shanmukhavadivu Subbulakshmi was an Indian Carnatic singer from Madurai, Tamil Nadu. She was the
first musician ever to be awarded the Bharat Ratna, India's highest civilian honour.

11) Consider the following statements regarding Hoysala architecture.


1. It was developed between the 11th and 14th centuries in southern Karnataka.
2. It has a unique style of blending elements of Dravida and Nagara architectures.
3. ‘Hoysala Temple’ form includes multiple shrines around a central pillared hall in a star-shaped layout.
4. They typically used Red Sandstone that allowed intricate carving of sculptures.
How many of the above statements is/are correct?
a) Only one
b) Only two
www.insightsonindia.com 28
INSTA STATIC QUIZ
c) Only three
d) All four

Solution: c)

Statement 4 is incorrect.

Hoysala architecture, developed between the 11th and 14th centuries in southern Karnataka under the Hoysala
Empire, is characterized by:
A unique style blending elements of Dravida and Nagara architectures.
Influences from Bhumija, Nagara, and Karntata Dravida architectural traditions.
Distinctive ‘Hoysala Temple’ form with multiple shrines around a central pillared hall in a star-shaped layout.
The use of soapstone, allowed intricate carving of sculptures, especially in the jewelry adorning temple walls.

12) Consider the following statements regarding Chausath Yogini temple.


1. It is located in Tamil Nadu.
2. It was built by King Devapala of the Kachchhapaghata dynasty.
2. It is believed that the structure of Chausath Yogini temple has inspired the architecture of old
parliament building of India.
4. The central shrine has a slab with perforations, for excess rainwater to drain off.
How many of the above statements is/are correct?
a) Only one
b) Only two
c) Only three
d) All four

Solution: c)

Statement 1 is incorrect.

The old Parliament building has for long been a symbol of India’s democracy, its graceful, dignified architecture
enshrining the jostling aspirations of a billion people.

However, a similar, round and pillared structure in India pre -dates the Parliament by several centuries, and many
believe it inspired the 20th century building. This structure is the Chausath Yogini temple in Mitaoli, Madhya
Pradesh.

According to the Morena district’s website, it was built around 1323 by King Devapala of the Kachchhapaghata
dynasty.
Dedicated to the 64 (chaunsath in Hindi) yoginis, its architecture is different from the temples dedicated to one
deity.

The Mitaoli temple is circular, with 64 chambers dedicated to the 64 yoginis, and a central shrine dedicated to
Shiva. While most Hindu temples have a shikhara, or projecting dome, the Mitaoli temple, like other Chausath
Yogini temples, is hypaethral, which means it has no roof. The central shrine has a slab with perforations, for
excess rainwater to drain off.

13) Kheer Bhawani Temple, recently in news is located in


a) Maharashtra
b) Uttarakhand
c) Odisha
d) Jammu and Kashmir

Solution: d)

www.insightsonindia.com 29
INSTA STATIC QUIZ
Kheer Bhawani temple is situated in Jammu and Kashmir.

14) Consider the following statements regarding Angkor Wat Temple.


1. Angkor Wat is a temple complex in Vietnam.
2. The temple is constructed in the Dravidian style and the sculptures depict episodes from the Ramayana
and the Mahabharata.
3. It was transformed into a Jain temple towards the end of the 12th century.
How many of the above statements is/are correct?
a) Only one
b) Only two
c) All three
d) None

Solution: a)

Only statement 2 is correct.

Angkor Wat is a temple complex in Cambodia and one of the largest religious monuments in the world.
It was originally constructed as a Hindu temple dedicated to the god Vishnu for the Khmer Empire, it was
gradually transformed into a Buddhist temple towards the end of the 12th century.
It was built by the Khmer King Suryavarman II in the early 12th century in Yaśodharapura (present-day Angkor),
the capital of the Khmer Empire, as his state temple and eventual mausoleum.
The temple is constructed in the Dravidian style and the sculptures depict episodes from the Ramayana and the
Mahabharata.

15) Consider the following statements regarding Konark Sun Temple.


1. It was built by King Narasimhadeva-I, the ruler of the Ganga dynasty.
2. It is designed in the form of a chariot mounted on 24 wheels.
3. The images of animals, foliage, dancing women and other structures are carved around the temple’s
base.
How many of the above statements is/are correct?
a) Only one
b) Only two
c) All three
d) None

Solution: c)

Built by King Narasimhadeva-I, the ruler of the Ganga dynasty, the Sun Temple is a symbol of Odisha’s glorious
heritage. Around 1,200 artisans and craftsmen took around 12 years to develop artistic magnificence and
engineering dexterity using chlorite and sandstone on the shores of the Bay of Bengal at Konark.
Since the ruler used to worship the sun, the temple was considered a chariot for the Sun God. Designed in the
form of a chariot mounted on 24 wheels, each of them is about 10 feet in diameter and drawn by seven mighty
horses. The images of animals, foliage, dancing women and other structures carved around the temple’s base
showcase the architectural grandeur.

While the main temple which enshrined the presiding deity and the Natya Mandap has fallen off and only the
remains can be seen, the large structure seen at present is known as the Jagamohana (the assembly hall or
audience hall or Mukhasala).
From 1900 to 1903, four entrances to the Jagamohana were sealed and filled with sand by the Britishers to
provide structural stability to the monument.

www.insightsonindia.com 30
INSTA STATIC QUIZ

5. History
1) Consider the following statements regarding Swadeshi Movement.
1. In 1905, the Boycott Resolution was passed at Mumbai’s historic Gowalia Tank Maidan and the formal
proclamation of Swadeshi Movement was made.
2. Newspapers such as Hitabadi, Sanjibani and Bengalee were used to propagate the agenda of Swadeshi
Movement.
Which of the above statements is/are incorrect?
a) 1 only
b) 2 only
c) Both 1 and 2
d) Neither 1 nor 2

Solution: a)

Anti-Partition Campaign under Moderates (1903-05):


In the period 1903-1905, the leadership was provided by Surendranath Banerjea, K.K. Mitra and Prithwish
chandra Ray. The methods adopted were petitions to the government, public meetings, memoranda, and
propaganda through pamphlets and newspapers such as Hitabadi, Sanjibani and Bengalee.

Their objective was to exert sufficient pressure on the government through an educated public opinion in India
and England to prevent the unjust partition of Bengal from being implemented.

Ignoring a loud public opinion against the partition proposal, the government announced partition of Bengal in
July 1905. Within days, protest meetings were held in small towns all over Bengal. It was in these meetings that
the pledge to boycott foreign goods was first taken. On August 7, 1905, with the passage of the Boycott
Resolution in a massive meeting held in the Calcutta Townhall, the formal proclamation of Swadeshi
Movement was made. After this, the leaders dispersed to other parts of Bengal to propagate the message of
boycott of Manchester cloth and Liverpool salt.

As a precursor to India’s independence, the Quit India Movement was launched at Mumbai’s historic Gowalia
Tank Maidan, now popularly known as August Kranti Maidan on 8th August 1942.

2) Which of the following was/were the recommendations of the Simon Commission setup in 1927?
1. India’s constitution should be unitary in nature.
2. Separate electorates should be abolished.
3. Elections to the legislative assemblies will be based on Universal adult franchise.
4. The provincial governments should devolve financial powers to the local bodies.
How many of the above statements is/are correct?
a) Only two
b) Only three
c) All four
d) None

Solution: d)

As per the commission, there should be a constitutional reconstruction in the form of a federal constitution. The
provinces should be given full autonomy including law.
Other major recommendations include:
• The number of members of provincial legislative council should be increased. Governor- general should
have complete power to appoint the members of the cabinet.
• The governor should have discretionary power to relate to internal security and administrative powers to
protect the different communities.
• The government of India should have complete control over the high court.
www.insightsonindia.com 31
INSTA STATIC QUIZ
• There were no Indian members in the commission. No universal franchise was proposed and the position
of governor-general remained unaffected.
• There was no provision to abolish separate electorate but it was rather extended to other communities
as well. No financial devolution was proposed.

3) Gandhiji represented the Indian National Congress in which of these Round Table Conferences?
a) First Round Table Conference, 1930-31
b) Second Round Table Conference, 1931
c) Third Round Table Conference, 1932
d) None of the above

Solution: b)

• The three Round Table Conferences of 1930–32 were a series of conferences organized by the British
Government and Indian national congress was participant to discuss constitutional reforms in India. These started
in November 1930 and ended in December 1932.
They were conducted as per the recommendation of Jinnah to Viceroy Lord Irwin and Prime Minister Ramsay
MacDonald, and by the report submitted by the Simon Commission in May 1930. Demands for swaraj, or self -
rule, in India had been growing increasingly strong.
• The Congress had boycotted the first conference.
• A settlement between Mahatma Gandhi and Viceroy Lord Irwin known as the Gandhi–Irwin Pact was reached
and Gandhi was appointed as the sole representative of the Congress to the second Round Table Conference.
Although MacDonald was still Prime Minister of Britain, he was by this time heading a coalition Government (the
“National Government”) with a Conservative majority. It was held in London in September 1931.

4) Consider the following statements regarding Young Bengal Movement.


1. The movement was launched by Surendranath Banerjea and later gained prominence with the
participation of Henry Vivian Derozio.
2. The movement had influence of Russian Revolution.
3. The movement failed to have a long-term impact.
How many of the above statements is/are correct?
a) Only one
b) Only two
c) All three
d) None

Solution: a)

Only Statement 3 is correct.

Young Bengal Movement was launched by Henry Louis Vivian Derozio (1809-1831), who had come to Calcutta in
1826 and was appointed in the Hindu College as a teacher.
• Drawing inspiration from the French Revolution, Derozio inspired his pupils to think freely and rationally,
question all authority, love liberty, equality and freedom, and oppose decadent customs and traditions. The
Derozians also supported women’s rights and education.
• The Derozians, however, failed to have a longterm impact. Derozio was removed from the Hindu College in
1831 because of his radicalism. The main reason for their limited success was the prevailing social conditions at
that time, which were not ripe for the adoption of radical ideas.
• Further, there was no support from any other social group or class.

5) Consider the following statements:


1. Factory Act, 1881 ruled that no children below the age of 14 years could be employed in a factory.
2. Full time factory inspectors were appointed under the Factory Act, 1911.

www.insightsonindia.com 32
INSTA STATIC QUIZ
3. The Whitley Commission on Labour was set up in 1929 to inquire into the existing conditions of labour
in industrial undertakings and plantations in India.
How many of the above statements is/are correct?
a) Only one
b) Only two
c) All three
d) None

Solution: b)

Statement 1 is incorrect.

Factories Act of I881 was the first legislation directly affecting industrial labourers in the country. It applied to all
manufacturing establishments using power driven machinery, employing 100 or more labours and working for
more than 120 days in a year. It ruled that no children below the age of 7 could be employed in a factory and
nobody before attaining the age of 12 could be made to work for more than 9 hours a day.
Full time factory inspectors were appointed under the Factory Act, 1911 to see whether the provisions were
being enforced.
The Royal Commission on Labour or the Whitley Commission on Labour was set up in 1929 to inquire into the
existing conditions of labour in industrial undertakings and plantations in India. The Commission recommended
the adoption of suitable legislation enabling the Competent Authority to collect and collate information regarding
the living, working and socio-economic conditions of industrial labour.

6) Consider the following statements regarding Defence of India Act 1915.


1. It was enacted by the Governor-General of India with the intention of curtailing the nationalist and
revolutionary activities in the aftermath of the First World War.
2. It made a sharp distinction between European and Indian subjects.
3. The act was first applied during the First Lahore Conspiracy trial in the aftermath of the Ghadar
Conspiracy.
How many of the above statements is/are correct?
a) Only one
b) Only two
c) All three
d) None

Solution: b)

Statement 2 is incorrect.

The Defence of India Act 1915, also referred to as the Defence of India Regulations Act, was an
emergency criminal law enacted by the Governor-General of India in 1915 with the intention of curtailing the
nationalist and revolutionary activities during and in the aftermath of the First World War.

Unlike the English law which was limited to persons of hostile associations or origin, the Defence of India act
could be applied to any subject of the King, and was used to an overwhelming extent against Indians.

The act was first applied during the First Lahore Conspiracy trial in the aftermath of the failed Ghadar
Conspiracy of 1915, and was instrumental in crushing the Ghadr movement in Punjab and the Anushilan Samiti in
Bengal.

7) Consider the following statements regarding the judicial reforms undertaken during the time of Warren
Hastings’ ascendancy?
1. Each district was provided with a civil court and a criminal court under the Collector.
2. The highest civil court of appeal was called Sadar Nizamat Adalat which was to function under an Indian
judge appointed by the Governor-in-Council.
www.insightsonindia.com 33
INSTA STATIC QUIZ
3. A digest of Hindu law was prepared in Sanskrit by learned Pandits.
How many of the above statements is/are correct?
a) Only one
b) Only two
c) All three
d) None

Solution: a)

Only Statement 3 is correct.

The judicial system at the time of Warren Hastings’ ascendancy was a store -house of abuses. The Nawab who was
hitherto the chief administrator of justice, misused his powers. Often, his judgments were careless. The zamindars
who acted as judges at lower levels within their own areas were highly corrupt and prejudiced. On the whole, the
judicial institution suffered from extreme corruption. Warren Hastings felt the necessity of reorganizing the
judicial system. Each district was provided with a civil court under the Collector and a criminal court under an
Indian Judge.

To hear appeals from the district courts two appellate courts, one for civil cases and another for criminal cases,
were established at Calcutta. The highest civil court of appeal was called Sadar Diwani Adalat, which was to be
presided over by the Governor and two judges recruited from among the members of his council. Similarly, the
highest appellate criminal court was known as Sadar Nizamat Adalat which was to function under an Indian
judge appointed by the Governor-in-Council.

Experts in Hindu and Muslim laws were provided to assist the judges. A digest of Hindu law was prepared in
Sanskrit by learned Pandits and it was translated into Persian. An English translation of it – Code of Hindu Laws
– was prepared by Halhed.

8) The events that were involved in and around the Tripuri Crisis of the Indian National Congress (INC) include
1. Leadership woes of the Indian National Congress (INC)
2. Controversies surrounding the merger of an arm of the All-India Muslim League with the Indian
National Congress (INC).
Select the correct answer code:
a) 1 only
b) 2 only
c) Both 1 and 2
d) Neither 1 nor 2

Solution: a)

In 1939 Congress Session was held in Tripuri. Tripuri is a small village in Madhya Pradesh. In this session of
Congress, Subhash Chandra Bose was elected for the Presidency of Indian National Congress. He won the
elections by securing 1580 votes defeating Sitaramayya who secured 1377 votes, according to the results that
were declared on 29 January, 1929. Mahatma Gandhi was not in favour of making Bose as President but he won
by majority of votes and became the President of Indian National Congress.

9) Consider the following statements regarding Deccan Riots.


1. The main motive of riots was to destroy the account books of the moneylender.
2. Deccan Agriculturists Relief Act was passed which ensured that the farmers could not be arrested and
imprisoned if they were unable to pay their debts.
3. The movement got support from the Poona Sarvajanik Sabha.
How many of the above statements is/are incorrect?
a) Only one
b) Only two
c) All three
www.insightsonindia.com 34
INSTA STATIC QUIZ
d) None

Solution: d)

In May and June 1875, peasants of Maharashtra in some parts of Pune and Ahmednagar districts revolted against
increasing agrarian distress. The Deccan Riots of 1875 targeted conditions of debt peonage (kamiuti) to
moneylenders. The rioters' specific purpose was to obtain and destroy the bonds, decrees, and other
documents in the possession of the moneylenders.

The movement also got support from the Poona Sarvajanik Sabha co-founded by M G Ranade.

It took several months for the police to restore order in the countryside. The Bombay government initially
dismissed the uprising as trivial. However, the Government of India pressurised Bombay to enquire into the
matter.
Accordingly, the Deccan Riots Commission was set up which presented a report to the British Parliament in 1878.
In 1879, the Agriculturists Relief Act was passed which ensured that the farmers could not be arrested and
imprisoned if they were unable to pay their debts.

10) Which of the following pairs of Newspaper and Editor are correctly matched?
1. Hitavada : Gopal Krishna Gokhale
2. Bengalee : Surendranath Banerjea
3. Mahratta : Bal Gangadhar Tilak
4. Voice of India : Bipin Chandra Pal
How many of the above options is/are correct?
a) Only one
b) Only two
c) Only three
d) All four

Solution: c)

Option 4 is incorrect.

Voice of India: Dadabhai Naoroji

11) According to the terms of the ‘Subsidiary Alliance’, Indian rulers were
1. Not allowed to have their independent armed forces
2. Supposed to protect the British representatives in the princely states
Select the correct answer code:
a) 1 only
b) 2 only
c) Both 1 and 2
d) Neither 1 nor 2

Solution: a)

According to the terms of this alliance, Indian rulers were not allowed to have their independent armed forces.
They were to be protected by the Company but had to pay for the “subsidiary forces” that the Company was
supposed to maintain for the purpose of this protection. If the Indian rulers failed to make the payment, then part
of their territory was taken away as penalty.
For example, when Richard Wellesley was Governor-General (1798-1805), the Nawab of Awadh was forced to
give over half of his territory to the Company in 1801, as he failed to pay for the “subsidiary forces”. Hyderabad
was also forced to cede territories on similar grounds.

12) Consider the following statements regarding Mahalwari system.


www.insightsonindia.com 35
INSTA STATIC QUIZ
1. Warren Hastings was associated with the conception of this system.
2. Under the system, ryots paid a variable sum to the Zamindars who then paid to the British.
3. Village lands, forestland and pastures were included under the system.
How many of the above statements is/are incorrect?
a) Only one
b) Only two
c) All three
d) None

Solution: b)

Only Statement 3 is correct.

In the North Western Provinces of the Bengal Presidency (most of this area is now in Uttar Pradesh), an
Englishman called Holt Mackenzie devised the new system which came into effect in 1822. He felt that the village
was an important social institution in north Indian society and needed to be preserved. Under his directions,
collectors went from village to village, inspecting the land, measuring the fields, and recording the customs and
rights of different groups. The estimated revenue of each plot within a village was added up to calculate the
revenue that each village (mahal) had to pay. This demand was to be revised periodically, not permanently fixed.
The charge of collecting the revenue and paying it to the Company was given to the village headman, rather
than the zamindar. This system came to be known as the mahalwari settlement.

The land included under this system consisted of all land of the villages, even the forestland, pastures etc.

13) Welby Commission setup in British India concerned an enquiry into the
a) Royal Indian Navy (RIN) mutiny
b) Atrocities of the Jallianwala Bagh massacre
c) Educational reforms in Indian Universities
d) Drain of wealth issue

Solution: d)

Dadabhai Naoroji in his famous book Poverty and UnBritish Rule in India wrote his Drain Theory.
He showed how India’s wealth was going away to England in the form of: (a) salaries, (b)savings, (c) pensions, (d)
payments to British troops in India and (e) profits of the British companies.
The British Government was forced to appoint the Welby Commission, with Dadabhai as the first Indian as its
member, to enquire into the Matter.
The Welby Commission's report, published in 1900, showed a number of cases where excessive or unjust
payments had been made by the Indian government.

14) The Poligar revolt of early 19th Century was rooted in


a) Enmity of tribals with non-tribals
b) Oppressive land revenue system
c) Control of tribal forests and river streams by British government
d) General discontentment with foreign rule

Solution: b)

The Polygar Wars or Palaiyakkarar Wars were fought between the Polygars of the former Tirunelveli Kingdom in
Tamil Nadu and the British East India Company forces between 1799 to 1805 over pending taxes, oppressive land
revenue system etc.
The British finally won after carrying out gruelling protracted jungle campaigns against the Polygar armies and
finally defeated them.
The British victory over the Polygars brought large parts of the territories of Tamil Nadu under British control,
enabling them to get a strong hold in Southern India.
www.insightsonindia.com 36
INSTA STATIC QUIZ

15) Which of the following factors influenced the growth of Modern Nationalism during British rule in India?
1. French Revolution.
2. Indian Renaissance.
3. Offshoot of modernization initiated by the British in India.
4. Strong reaction to British imperialist policies in India.
How many of the above options is/are correct?
a) Only one
b) Only two
c) Only three
d) All four

Solution: d)

The rise and growth of Indian nationalism has been traditionally explained in terms of Indian response to the
stimulus generated by the British Raj through creation of new institutions, new opportunities, resources, etc. In
other words, Indian nationalism grew partly as a result of colonial policies and partly as a reaction to colonial
policies. In fact, it would be more correct to see Indian nationalism as a product of a mix of various factors:
• Worldwide upsurge of the concepts of nationalism and right of self -determination initiated by the French
Revolution.
• Indian Renaissance.
• Offshoot of modernisation initiated by the British in India.
• Strong reaction to British imperialist policies in India.

6. Environment
1) Consider the following statements regarding Community Reserve
1. Community Reserve is established under Environment Protection Act, 1986.
2. Any state government can declare community-owned forest area as “community reserve” if locals are
willing to participate in conservation efforts for same.
3. Community reserve receives same legal protection as National Parks and Wildlife Sanctuaries.
How many of the above statements is/are correct?
a) Only one
b) Only two
c) All three
d) None

Solution: b)

Statement 1 is incorrect.

Community Reserves:
• Conservation reserves and community reserves in India are terms denoting protected areas of India which
typically act as buffer zones to or connectors and migration corridors between established national parks, wildlife
sanctuaries and reserved and protected forests of India.
• Such areas are designated as conservation areas if they are uninhabited and completely owned by the
Government of India but used for subsistence by communities and community areas if part of the lands are
privately owned.
• These protected area categories were first introduced in the Wildlife (Protection) Amendment Act of 2002 −
the amendment to the Wildlife Protection Act of 1972.

www.insightsonindia.com 37
INSTA STATIC QUIZ
• It enables any state government to declare community-owned forest area as “community reserve” if locals
are willing to participate in conservation efforts for same.
• Under it, community reserve receives same legal protection as National Parks and Wildlife Sanctuaries.

2) Consider the following statements regarding Reserved Forests.


1. The term Reserved Forest was first introduced in the Environment (Protection) Act, 1986.
2. These are constituted by the State Government
3. Generally In reserved forests local people are prohibited for settlement.
How many of the above statements is/are correct?
a) Only one
b) Only two
c) All three
d) None

Solution: b)

Statement 1 is incorrect.

Reserved Forests:
Reserve forests are the most restricted forests and are constituted by the State Government on any forest land
or wasteland which is the property of the Government.
In reserved forests, local people are prohibited, unless specifically allowed by a Forest Officer in the course of the
settlement.
The term was first introduced in the Indian Forest Act, 1927 in British India, to refer to certain forests granted
protection under the British crown in British India, but not associated suzerainty.

3) Consider the following statements.


1. Nilgiri biosphere reserve is the largest protected forest area in India.
2. Nilgiri biosphere reserve is spread across Tamil Nadu, Karnataka, Kerala and Telangana.
3. India's natural Gold fields are located in the regions in and around Nilgiris Biosphere Reserve.
How many of the above statements is/are correct?
a) Only one
b) Only two
c) All three
d) None

Solution: b)

Statement 2 is incorrect.

Nilgiri biosphere reserve is the largest protected forest area in India, spreading across Tamil Nadu, Karnataka
and Kerala.
India's natural Gold fields are also located in the regions in and around Nilgiris Biosphere Reserve scattered in
the states of Karnataka, Kerala, and Tamil Nadu.

4) Joint Forest Management Committee (JFMC) is an agency formed at a village level or a cluster of villages
situated adjacent to Reserved Forests (RF). Which of the following are the functions of Joint Forest
Management Committee (JFMC)?
1. Assure employment opportunities to the tribal communities.
2. Ensure sustainable management of forest resources
3. Selecting the plant species to be planted in the forest
4. Restore watershed capability in catchment areas
How many of the above statements is/are correct?
a) Only one
b) Only two
www.insightsonindia.com 38
INSTA STATIC QUIZ
c) Only three
d) All four

Solution: d)

Joint Forest Management Committee (JFMC) is an agency formed at a village level or a cluster of villages
situated adjacent to Reserved Forests (RF) registered with the Territorial Divisional Forest Office.

JFMC is responsible for selecting the plant species to be planted in the forest, suggesting physical and financial
targets, and conducting awareness programs. The JFMC objectives are to ensure sustainable management of
forest resources, to improve forest cover via afforestation, to restore degraded forest land, to promote
conservation awareness through environmental education, restore watershed capability in catchment areas and
to assure employment opportunities to the tribal communities.

5) Consider the following statements.


1. Global Assessment Report on Biodiversity and Ecosystem Services was released by
the Intergovernmental Science-Policy Platform on Biodiversity and Ecosystem Services (IPBES).
2. The World Network of Biosphere Reserves is created by UNEP.
3. The Nilgiri Biosphere Reserve is designated as the first biosphere reserve in India.
How many of the above statements are correct?
a) Only one
b) Only two
c) All three
d) None

Solution: b)

Statement 2 is incorrect.

Global Assessment Report on Biodiversity and Ecosystem Services released by the Intergovernmental Science-
Policy Platform on Biodiversity and Ecosystem Services (IPBES).

World Network of Biosphere Reserves, created in 1971 by UNESCO.

The Nilgiri Biosphere Reserve was the first biosphere reserve in India designated in the year 1986 by UNESCO. It
stretches across the States of Tamil Nadu, Karnataka and Kerala.

6) Consider the following statements.


1. Climate change and unprecedented floods can facilitate the introduction of aquatic invasive alien
species into new habitats.
2. Invasive alien species slowly begin to wipe out local diversity and the economy by altering the functions
of the ecosystem.
Which of the above statements is/are incorrect?
a) 1 only
b) 2 only
c) Both 1 and 2
d) Neither 1 nor 2

Solution: d)

Climate change and unprecedented floods resulting from cloud bursts have facilitated the introduction of
aquatic invasive alien species into new habitats in India. This phenomenon threatens ecosystems, habitats and
native species.

www.insightsonindia.com 39
INSTA STATIC QUIZ
Researchers say that during heavy floods, invasive alien fishes which are illegally farmed in fragile systems,
including domestic aquarium tanks, ponds, lakes and abandoned quarries, effortlessly escape from captivity and
enter nearby wetlands. After a while, they slowly begin to wipe out local diversity and the economy by altering
the functions of the ecosystem.

7) Which of the following statements is correct regarding ‘Bamboonomics’?


a) Encourage bamboo plantation by farmers in private lands.
b) Promoting Bamboo as an alternative in building construction.
c) Bamboo plantation in urban areas in order to maintain a sustainable environment and pollution free
clean air.
d) A movement to combat desertification and climate change

Solution: d)

Bamboonomics:
It is a movement to combat desertification and climate change which will involve the tribal community of
India since they have the expertise in this field.
The movement will ensure that tribals can earn a livelihood without causing environmental harm.
It was launched at the COP 14 of the United Nations Convention to Combat Desertification (UNCCD).

8) Consider the following statements regarding Ocean deoxygenation.


1. Ocean deoxygenation is the maximum reduction of oxygen minimum zones in the world's oceans as a
consequence of anthropogenic emissions of carbon dioxide.
2. Oxygen is less soluble in warmer water.
3. The areas of Sea water that is depleted of dissolved oxygen is called Anoxic waters.
How many of the above statements is/are correct?
a) Only one
b) Only two
c) All three
d) None

Solution: b)

Statement 1 is incorrect.

Ocean deoxygenation is the expansion of oxygen minimum zones in the world's oceans as a consequence
of anthropogenic emissions of carbon dioxide. The change has been fairly rapid and poses a threat to fish and
other types of marine life, as well as to people who depend on marine life for nutrition or livelihood.

Ocean deoxygenation poses implications for ocean productivity, nutrient cycling, carbon cycling, and marine
habitats.

Most of the excess heat from CO 2 and other greenhouse gas emissions is absorbed by the oceans. Warmer oceans
cause deoxygenation both because oxygen is less soluble in warmer water, and through temperature driven
stratification of the ocean which inhibits the production of oxygen from photosynthesis.

Anoxic waters are areas of sea water, fresh water, or groundwater that are depleted of dissolved oxygen and
are a more severe condition of hypoxia.

9) During winter the vehicles with closed doors and windows remain warmer than the temperature outside. This
may be due to
a) High pollution outside the vehicle
b) Heat Conduction by metallic frame of vehicle
c) Ionizing radiation effect

www.insightsonindia.com 40
INSTA STATIC QUIZ
d) Greenhouse effect

Solution: d)

A greenhouse is made up of glass. The glass which is transparent to incoming short wave solar radiation is opaque
to outgoing long wave radiation. The glass, therefore, allows in more radiation and prevents the long wave
radiation going outside the glass house, causing the temperature inside the glasshouse structure warmer than
outside.
When you enter a car or a bus, during summers, where windows are closed, you feel more heat than outside.
Likewise during winter the vehicles with closed doors and windows remain warmer than the temperature
outside.

10) Consider the following statements regarding ocean acidification.


1. It largely occurs due to the high absorption of nitrogenous based acidic compounds.
2. Introduction of sea grasses can reduce the impact of acidification.
Which of the above statements is/are incorrect?
a) 1 only
b) 2 only
c) Both 1 and 2
d) Neither 1 nor 2

Solution: a)

Ocean acidification refers to a reduction in the pH of the ocean over an extended period of time, caused
primarily by uptake of carbon dioxide (CO2) from the atmosphere. Nitrogenous compounds contribute
fraction to ocean acidification.

Sea grass has ability to control ocean acidification.

11) Consider the following statements regarding Kelp forests.


1. Kelp forests are underwater areas which provide food and shelter for fish, invertebrates and marine
mammal species.
2. They offer crucial services such as carbon sequestration and erosion control.
3. Bryozoa are highly beneficial for increasing the productivity of kelp forests.
How many of the above statements is/are correct?
a) Only one
b) Only two
c) All three
d) None

Solution: b)

Statement 3 is incorrect.

Kelps cover 25 percent of the world’s coastlines and provide food and shelter for fish, invertebrates and marine
mammal species. They also offer crucial services such as carbon sequestration and erosion control, according to
scientists.
But kelps are increasingly threatened by climate change, eutrophication and shoreline development, among other
human-induced stressors.
One such threat is from bryozoa, moss animals that grow as mats on kelps. They drive the seaweed to sink into
the seafloor and disintegrate.

The outbreak can be linked to high temperatures. For example, in 2015, researchers observed an expansive
outbreak of bryozoa. These observations coincided with ‘the blob’, a marine heat wave that emerged across the
northeast Pacific Ocean, according to a report.
www.insightsonindia.com 41
INSTA STATIC QUIZ

Dense kelp beds in warmer and less wave-exposed sites are more susceptible to bryozoan outbreaks.

Seaweeds are farmed as they have industrial applications. They can be harvested to extract biostimulants for
plant growth. They also can help in long-term carbon sequestration and regeneration of ocean habitats.

12) There are many documented cases of groundwater pollution which followed coal ash disposal. Which of these
elements in coal ash makes it so harmful?
1. Arsenic
2. Cadmium
3. Mercury
4. Sorbitol
How many of the above options is/are correct?
a) Only one
b) Only two
c) Only three
d) All four

Solution: c)

Option 4 is incorrect.

Since coal contains trace levels of trace elements (like e.g. arsenic, barium, beryllium, boron, cadmium,
chromium, thallium, selenium, molybdenum and mercury).
Fly ash obtained after combustion of this coal contains enhanced concentrations of these elements, and therefore
the potential of the ash to cause groundwater pollution is high.

13) Consider the following statements regarding Solid wastes to energy plants.
1. Waste-to-energy plants can convert salt water to potable fresh water.
2. The Processing of waste into biofuel is carbon-negative.
3. The residue ash can be used in manufacturing cinder blocks or for road construction.
How many of the above statements is/are correct?
a) Only one
b) Only two
c) All three
d) None

Solution: c)

Benefits of wastes to energy plants:


• In terms of volume, usually waste-to-energy plants incinerate 80 to 90 percent of waste, thus helping large
cities from choking due to unmanageable waste.
• Waste to energy generates clean, reliable energy from a renewable fuel source, thus reducing dependence on
fossil fuels, the combustion of which is a major contributor to Greenhouse Gas (GHG) emissions.
• Sometimes, the residue ash is clean enough to be used for some purposes such as raw materials for use in
manufacturing cinder blocks or for road construction.
• Some waste-to-energy plants convert salt water to potable fresh water as a by-product of cooling
processes.
• Waste-to-energy plants cause less air pollution than coal plants.
• It is carbon-negative – processing waste into biofuel releases considerably less carbon and methane into the
air than having waste decay away in landfills or the lake.

14) Consider the following statements regarding Gas Hydrates

www.insightsonindia.com 42
INSTA STATIC QUIZ
1. Gas hydrates are formed when a gas such as methane gets trapped in well-defined cages of water
molecules forming crystalline solids.
2. Their decomposition can release large amounts of Carbon Dioxide (CO2) that could impact Earth’s
climate.
3. Sudden release of pressurized methane gas from ocean base may cause submarine landslides, which in
turn can trigger tsunamis.
How many of the above statements is/are correct?
a) Only one
b) Only two
c) All three
d) None

Solution: b)

Statement 2 is incorrect.

Gas hydrates are ice-like crystalline minerals that form when low molecular weight gas (such as methane,
ethane, or carbon dioxide) combines with water and freezes into a solid under low temperature and moderate
pressure conditions.
• Most gas hydrates are formed from methane (CH4), which has led to the terms “gas hydrate” and “methane
hydrate” often being used interchangeably.
• On Earth, gas hydrates occur naturally in some marine sediments and within and beneath permafrost. They are
also speculated to form on other planets.

Hydrate deposits are important for a variety of reasons:


• Gas hydrate deposits may contain roughly twice the carbon contained in all reserves of coal, oil, and
conventional natural gas combined, making them a potentially valuable energy resource.
• Their decomposition can release large amounts of methane, which is a greenhouse gas that could impact
Earth’s climate.
• Sudden release of pressurized methane gas may cause submarine landslides, which in turn can trigger
tsunamis.

15) Consider the following statements.


1. Decomposition is the process where decomposers break down complex organic matter into inorganic
substances like carbon dioxide, water and nutrients.
2. Catabolism is the process where the humus is further degraded by some microbes and release of
inorganic nutrients occur.
3. Mineralization is process where bacterial and fungal enzymes degrade detritus into simpler inorganic
substances.
How many of the above statements is/are incorrect?
a) Only one
b) Only two
c) All three
d) None

Solution: b)

Only Statement 1 is correct.

You may have heard of the earthworm being referred to as the farmer’s ‘friend’. This is so because they help in
the breakdown of complex organic matter as well as in loosening of the soil. Similarly, decomposers break down
complex organic matter into inorganic substances like carbon dioxide, water and nutrients and the process is
called decomposition. Dead plant remains such as leaves, bark, flowers and dead remains of animals, including
fecal matter, constitute detritus, which is the raw material for decomposition. The important steps in the process
of decomposition are fragmentation, leaching, catabolism, humification and mineralisation.
www.insightsonindia.com 43
INSTA STATIC QUIZ
• Detritivores (e.g., earthworm) break down detritus into smaller particles. This process is called
fragmentation.
• By the process of leaching, water soluble inorganic nutrients go down into the soil horizon and get
precipitated as unavailable salts.
• Bacterial and fungal enzymes degrade detritus into simpler inorganic substances. This process is called as
catabolism.
• It is important to note that all the above steps in decomposition operate simultaneously on the detritus.
Humification and mineralisation occur during decomposition in the soil. Humification leads to accumulation
of a dark coloured amorphous substance called humus that is highly resistant to microbial action and
undergoes decomposition at an extremely slow rate. Being colloidal in nature it serves as a reservoir of
nutrients. The humus is further degraded by some microbes and release of inorganic nutrients occur by the
process known as mineralisation.

16) Consider the following statements.


1. The World Environment Day, led by International Union for Conservation of Nature (IUCN) has been
held annually on June 5.
1. The date for World Environment Day was chosen during the 1972 Stockholm Conference on the Human
Environment.
3. Stockholm Conference on the Human Environment is considered to be the first world conference to
make the environment a major issue.
How many of the above statements are correct?
a) Only one
b) Only two
c) All three
d) None

Solution: b)

Statement 1 is incorrect.

The World Environment Day, led by the United Nations Environment Programme (UNEP), has been held
annually on June 5, since 1973. The date was chosen by the UN General Assembly during the historic 1972
Stockholm Conference on the Human Environment – considered to be the first world conference to make the
environment a major issue.

17) Consider the following pairs.


1. National Mission for a Green India: It aims to increase forest cover on degraded lands and protect
existing forested lands.
2. Green Credit Programme: The objective is to incentivize environmentally sustainable and responsive
actions by companies, individuals and local bodies.
3. MISHTI Scheme: Reducing inputs of synthetic fertilizers and pesticides for sustaining agriculture.
How many of the above statements is/are incorrect?
a) Only one
b) Only two
c) All three
d) None

Solution: a)

Statement 3 is incorrect.

The National Mission for a Green India aims to increase forest cover on degraded lands and protect existing
forested lands.

www.insightsonindia.com 44
INSTA STATIC QUIZ
The Green Credit Programme has the objective to “incentivize environmentally sustainable and responsive
actions by companies, individuals and local bodies”.

The Mangrove Initiative for Shoreline Habitats & Tangible Incomes (MISHTI) is particularly significant because of
the extraordinary importance of mangroves and coastal ecosystems in mitigating climate change.

The Prime Minister Programme for Restoration, Awareness, Nourishment, and Amelioration of Mother Earth
(PM-PRANAM) for reducing inputs of synthetic fertilizers and pesticides is critical for sustaining our
agriculture.

Finally, the Amrit Dharohar scheme directly mentions our biological wealth and is expected to “encourage
optimal use of wetlands, and enhance biodiversity, carbon stock, eco-tourism opportunities and income
generation for local communities”.

18) Consider the following statements.


1. The Tamil Nadu government launched Nilgiri Tahr conservation project, an initiative for the
conservation of the Nilgiri Tahr.
2. Nilgiri Tahr is an endangered species, endemic to western ghats.
3. There are multiple references to the Nilgiri Tahr in Tamil Sangam literature.
How many of the above statements is/are correct?
a) Only one
b) Only two
c) All three
d) None

Solution: c)

Tamil Nadu government launched Nilgiri Tahr conservation project, an initiative for the conservation of the
Nilgiri Tahr, the State animal.

Under the Nilgiri Tahr project, the government plans to develop a better understanding of the Nilgiri Tahr
population through surveys and radio telemetry studies; reintroduce the Tahrs to their historical habitat; address
proximate threats; and increase public awareness of the species. The project is to be implemented from 2022 to
2027.

According to the order, there are multiple references to the Nilgiri Tahr in Tamil Sangam literature dating back
to 2,000 years. The late Mesolithic (10,000-4,000 BC) paintings highlight the significance of the Tahr in folklore,
culture and life. It was designated as the State animal in recognition of its ecological and cultural significance.
The animal is endemic to the Western Ghats. It is protected under Schedule-I of the Wildlife (Protection) Act of
India, 1972.

It is an endangered species and the sole Caprinae species found in the tropical mountains of southern India. The
animal inhabits meadows with steep cliffs at elevations between 300 metres and 2,600 metre above sea level.

19) Consider the following statements regarding Ground-level ozone.


1. Ground-level ozone is a colourless gas that forms up to 2 miles above the ground.
2. Ground-level ozone is called a “secondary” pollutant, since it is produced when two primary pollutants
react in sunlight and stagnant air.
3. It is highly stationary and does not travel long distances due to wind.
How many of the above statements are incorrect?
a) Only one
b) Only two
c) All three
d) None

www.insightsonindia.com 45
INSTA STATIC QUIZ
Solution: a)

Statement 3 is incorrect.

What is ground-level ozone?


Also known as tropospheric ozone, ground-level ozone is a colourless and highly irritating gas that forms just
above the Earth’s surface (up to 2 miles above the ground).

Notably, it’s not directly emitted into the air but rather produced when two primary pollutants react in sunlight
and stagnant air. These two primary pollutants are nitrogen oxides (NOx) and volatile organic compounds (VOCs).
Therefore, ground-level ozone is called a “secondary” pollutant.

Ground-level ozone is likely to breach safety standards on hot summer days in urban areas, but can also reach
unhealthy levels during colder months. The pollutant can also travel long distances due to wind and affect rural
areas also.

20) Sharks are ectotherms. What does it mean?


a) If heat loss exceeds heat generation, metabolism increases to make up the loss
b) They maintain a constant body temperature independent of the environment
c) Their body temperature is largely determined by the surrounding water temperature
d) None of the above

Solution: c)

Ectotherm, any so-called cold-blooded animal—that is, any animal whose regulation of
body temperature depends on external sources, such as sunlight or a heated rock surface. The ectotherms
include the fishes, amphibians, reptiles, and invertebrates. The body temperature of an aquatic ectotherm is
usually very close to the temperature of the surrounding water.

21) Consider the following statements regarding International Union for Conservation of Nature (IUCN).
1. Its main aim is to mobilize the public in support of nature conservation.
2. It was involved in establishing the World-Wide Fund for Nature.
3. It has observer and consultative status at the United Nations.
How many of the above statements is/are correct?
a) Only one
b) Only two
c) All three
d) None

Solution: b)

Statement 1 is incorrect.

The International Union for Conservation of Nature (IUCN) is an international organization working in the field
of nature conservation and sustainable use of natural resources. It is involved in data gathering and analysis,
research, field projects, advocacy, and education. IUCN's mission is to "influence, encourage and assist societies
throughout the world to conserve nature and to ensure that any use of natural resources is equitable and
ecologically sustainable".

Over the past decades, IUCN has widened its focus beyond conservation ecology and now incorporates issues
related to sustainable development in its projects. IUCN does not itself aim to mobilize the public in support of
nature conservation. It tries to influence the actions of governments, business and other stakeholders by
providing information and advice, and through building partnerships.

www.insightsonindia.com 46
INSTA STATIC QUIZ
The organization is best known to the wider public for compiling and publishing the IUCN Red List of Threatened
Species, which assesses the conservation status of species worldwide.

IUCN has observer and consultative status at the United Nations and plays a role in the implementation of
several international conventions on nature conservation and biodiversity. It was involved in establishing
the World Wide Fund for Nature and the World Conservation Monitoring Centre.

22) Consider the following statements regarding UNFCCC.


1. The UNFCCC established a framework for action to stabilise concentrations of greenhouse gases in the
earth’s atmosphere.
2. Nearly all member states of the United Nations have ratified the convention.
3. The framework sets binding limits on greenhouse gas emissions for individual countries.
How many of the above statements is/are correct?
a) Only one
b) Only two
c) All three
d) None

Solution: b)

Statement 3 is incorrect.

The UNFCCC was adopted in 1992 at the Rio Earth Summit, which marked the beginning of the international
community’s first concerted effort to confront the problem of climate change.
Known also as the Rio Convention, the UNFCCC established a framework for action to stabilise concentrations
of greenhouse gases in the earth’s atmosphere.
The UNFCCC entered into force in 1994, and nearly all of the world’s nations have ratified (By 2022, the UNFCCC
had 198 parties).
The framework sets non-binding limits on greenhouse gas emissions for individual countries and contains no
enforcement mechanisms.

23) Consider the following statements regarding IUCN Red List.


1. The aim of the IUCN Red List is to convey the urgency of conservation issues to the public and policy
makers, as well as help the international community to reduce species extinction.
2. It is the world’s most comprehensive inventory of the global conservation status of biological species.
3. The pink pages in this publication include the formerly endangered species.
How many of the above statements is/are correct?
a) Only one
b) Only two
c) All three
d) None

Solution: b)

Statement 3 is incorrect.

IUCN Red List or Red Data List or Red Book


• The IUCN Red List of Threatened Species, founded in 1964, is the world’s most comprehensive inventory of
the global conservation status of biological species.
• The aim of the IUCN Red List is to convey the urgency of conservation issues to the public and policy makers,
as well as help the international community to reduce species extinction.
• The pink pages in this publication include the critically endangered species.
• As the status of the species changes, new pages are sent to the subscribers.

www.insightsonindia.com 47
INSTA STATIC QUIZ
• Green pages are used for those species that were formerly endangered but have now recovered to a point
where they are no longer threatened.
• With passing time, the number of pink pages continue to increase.

24) Which of the following are International Organization Partners of Ramsar Convention?
1. Mangrove Alliance for Climate (MAC)
2. Birdlife International
3. International Union for Conservation of Nature
4. WWF International
Select the correct answer code:
a) 1, 3, 4
b) 2, 3, 4
c) 1, 2, 3
d) 1, 2, 3, 4

Solution: b)

International organization partners:


The Ramsar Convention works closely with six other organisations known as international organization partners
(IOPs). These are:
• BirdLife International
• International Union for Conservation of Nature (IUCN)
• International Water Management Institute (IWMI)
• Wetlands International
• WWF International
• Wildfowl & Wetlands Trust (WWT)
These organizations support the work of the convention by providing expert technical advice, helping implement
field studies, and providing financial support. The IOPs also participate regularly as observers in all meetings of
the conference of the parties and as full members of the Scientific and Technical Review Panel.

25) Consider the following statements regarding Appendices covered under CITES.
1. Appendix I includes species not threatened with extinction.
2. Appendix II includes species that are necessarily threatened with extinction.
3. Appendix III contains species that are protected in at least one country, which has asked other CITES
Parties for assistance in controlling the trade.
How many of the above statements is/are incorrect?
a) Only one
b) Only two
c) All three
d) None

Solution: b)

Only Statement 3 is correct.

The species covered under CITES are listed in three Appendices, according to the degree of protection they need.

Appendix I includes species threatened with extinction. Trade in specimens of these species is permitted rarely,
only in “exceptional circumstances”, such as gorillas, and lions from India.

Appendix II includes species not necessarily threatened with extinction, but in which trade must be controlled to
ensure their survival. For example, certain kinds of foxes and Hippopotamuses.

www.insightsonindia.com 48
INSTA STATIC QUIZ
Appendix III contains species that are protected in at least one country, which has asked other CITES Parties for
assistance in controlling the trade, like the Bengal fox or the Golden Jackal from India. Different procedures are
given category-wise to engage in the trade of species in each of the lists.

26) Each organism has an invariably defined range of conditions that it can tolerate, diversity in the resources
it utilises and a distinct functional role in the ecological system, all these together comprise its
a) Ecology
b) Ecosystem
c) Ecological Niche
d) Ecotone

Solution: c)

What are the key elements that lead to so much variation in the physical and chemical conditions of different
habitats?
The most important ones are temperature, water, light and soil. We must remember that the physico -chemical
(abiotic) components alone do not characterize the habitat of an organism completely; the habitat includes biotic
components also – pathogens, parasites, predators and competitors – of the organism with which they interact
constantly. We assume that over a period of time, the organism had through natural selection, evolved
adaptations to optimise its survival and reproduction in its habitat. Each organism has an invariably defined range
of conditions that it can tolerate, diversity in the resources it utilises and a distinct functional role in the ecological
system, all these together comprise its niche.

27) Consider the following statements.


1. In a terrestrial ecosystem, a much larger fraction of energy flows through the grazing food chain than
through the detritus food chain.
2. In an aquatic ecosystem, detritus food chain is the major conduit for energy flow.
3. Detritus food chain may be connected with the grazing food chain at some levels.
How many of the above statements is/are correct?
a) Only one
b) Only two
c) All three
d) None

Solution: a)

Only Statement 3 is correct.

In an aquatic ecosystem, Grazing food chain is the major conduit for energy flow.

As against this, in a terrestrial ecosystem, a much larger fraction of energy flows through the detritus food chain
than through the grazing food chain.

Detritus food chain may be connected with the grazing food chain at some levels: some of the organisms of DFC
are prey to the GFC animals, and in a natural ecosystem, some animals like cockroaches, crows, etc., are
omnivores.
These natural interconnection of food chains make it a food web.

28) Consider the following statements.


1. Ecological Succession is the gradual process by which ecosystems change and develop over time.
2. Hydrarch succession takes place in wetter areas and xerarch succession takes place in dry areas.
3. The species that invade a bare area are called keystone species.
How many of the above statements is/are correct?
a) Only one
b) Only two
www.insightsonindia.com 49
INSTA STATIC QUIZ
c) All three
d) None

Solution: b)

Statement 3 is incorrect.

Ecological Succession is the gradual process by which ecosystems change and develop over time.
Based on the nature of the habitat – whether it is water (or very wet areas) or it is on very dry areas – succession
of plants is called hydrach or xerarch, respectively.

Hydrarch succession takes place in wetter areas and the successional series progress from hydric to the mesic
conditions.
As against this, xerarch succession takes place in dry areas and the series progress from xeric to mesic conditions.
Hence, both hydrarch and xerach successions lead to medium water conditions (mesic) – neither too dry (xeric)
nor too wet (hydric).

The species that invade a bare area are called pioneer species. In primary succession on rocks these are usually
lichens which are able to secrete acids to dissolve rock, helping in weathering and soil formation.

A keystone species is a species that has a disproportionately large effect on its natural environment relative to its
abundance. Without keystone species, the ecosystem would be dramatically different or cease to exist
altogether.

29) Consider the following statements regarding Ecological Succession.


1. The gradual and fairly predictable change in the species composition of a given area is called ecological
succession.
2. Primary succession begins in areas where natural biotic communities have been destroyed.
3. The entire sequence of communities that successively change in a given area are called sere(s).
How many of the above statements is/are correct?
a) Only one
b) Only two
c) All three
d) None

Solution: b)

Statement 2 is incorrect.

ECOLOGICAL SUCCESSION
• An important characteristic of all communities is that their composition and structure constantly change in
response to the changing environmental conditions. This change is orderly and sequential, parallel with the
changes in the physical environment. These changes lead finally to a community that is in near equilibrium with
the environment and that is called a climax community. The gradual and fairly predictable change in the species
composition of a given area is called ecological succession. During succession some species colonies an area and
their population become more numerous whereas populations of other species decline and even disappear.
• The entire sequence of communities that successively change in a given area are called sere(s). The individual
transitional communities are termed seral stages or seral communities.
• Succession is hence a process that starts in an area where no living organisms are there – these could be areas
where no living organisms ever existed, say bare rock; or in areas that somehow, lost all the living organisms that
existed there. The former is called primary succession, while the latter is termed secondary succession.
• Secondary succession begins in areas where natural biotic communities have been destroyed such as in
abandoned farm lands, burned or cut forests, lands that have been flooded. Since some soil or sediment is
present, succession is faster than primary succession.

www.insightsonindia.com 50
INSTA STATIC QUIZ
30) Molds derive energy through
a) Photosynthesis
b) Heterotrophy
c) Sulphur synthesis
d) Autotrophy

Solution: b)

We can find molds on degraded bread. Like all fungi, molds derive energy not through photosynthesis but from
the organic matter on which they live, utilizing heterotrophy. Typically, molds secrete hydrolytic enzymes, mainly
from the hyphal tips. These enzymes degrade complex biopolymers such as starch, cellulose and lignin into
simpler substances which can be absorbed by the hyphae. In this way, molds play a major role in causing
decomposition of organic material, enabling the recycling of nutrients throughout ecosystems.

31) Consider the following statements regarding the distribution of Wetlands in India.
1. According to National Wetland Inventory and Assessment compiled by ISRO, wetlands are spread
over more than 10 percent of the total geographical area of India.
2. In state-wise distribution of wetlands, West Bengal is at the top position.
3. Inland-natural wetlands account for maximum share compared to coastal-natural wetlands.
How many of the above statements is/are correct?
a) Only one
b) Only two
c) All three
d) None

Solution: a)

Only Statement 3 is correct.

Globally, wetlands cover 6.4 per cent of the geographical area of the world. In India, according to the National
Wetland Inventory and Assessment compiled by the Indian Space Research Organisation (ISRO), wetlands are
spread over 4.63 per cent of the total geographical area of the country. Of this Inland-natural wetlands account
for 43.4% and coastal-natural wetlands 24.3%.

In state-wise distribution of wetlands, Gujarat is at the top (17.56% of total geographical area of the state), or
22.7% of total wetlands areas of the country thanks to a long coastline. It is followed by Andhra Pradesh, Uttar
Pradesh and West Bengal.

32) Which of the following can reveal patterns of climate change on earth?
a) Study of Interglacial periods
b) Study of tree rings
c) Both a and b
d) Neither a nor b

Solution: c)

An interglacial period (or alternatively interglacial) is a geological interval of warmer global average temperature
lasting thousands of years that separates consecutive glacial periods within an ice age.
Evidence of multiple advances and retreats of glaciers, and the sediment deposits in glacial lakes reveal the
occurrence of warm and cold periods in the history of earth. It is a strong evidence of climate change.

Because trees are sensitive to local climate conditions, such as rain and temperature, they give scientists some
information about that area's local climate in the past. For example, tree rings usually grow wider in warm, wet
years and they are thinner in years when it is cold and dry.

www.insightsonindia.com 51
INSTA STATIC QUIZ
33) Consider the following statements regarding Ruminant species.
1. Ruminants are herbivorous grazing mammals that are able to acquire nutrients from plant-based food
by fermenting it in a specialized stomach prior to digestion, principally through microbial actions.
2. Cows, sheep, goats, and buffaloes are some of the Ruminants.
3. Ruminant animals release methane mainly through burping.
How many of the above statements is/are correct?
a) Only one
b) Only two
c) All three
d) None

Solution: c)

Ruminant species are hooved grazing or browsing herbivores that chew cud. Ruminants such as cows, sheep,
goats, and buffaloes have a special type of digestive system that allows them to break down and digest food
that non-ruminant species would be unable to digest.

Stomachs of ruminant animals have four compartments, one of which, the rumen, helps them to store partially
digested food and let it ferment. This partially digested and fermented food is regurgitated by the animals who
chew through it again and finish the digestive process.

However, as grass and other vegetation ferments in the rumen, it generates methane, a potent greenhouse gas.
Ruminant animals such as cows and sheep release this methane mainly through burping.
Given the very large numbers of cattle and sheep on farms in dairy-producing countries, these emissions add up
to a significant volume. It is estimated that the ruminant digestive system is responsible for 27 per cent of all
methane emissions from human activity.

34) Consider the following statements.


1. In its raw form, biogas can be used as a clean cooking fuel.
2. Biogas can be purified and upgraded up to 98 per cent of purity to make it suitable to be used as a
green fuel for transportation or filling of cylinders.
3. The Sustainable Alternatives Towards Affordable Transporation (SATAT) scheme by the Ministry of New
and Renewable Energy encourages entrepreneurs to set up compressed biogas (CBG) plants.
How many of the above statements is/are correct?
a) Only one
b) Only two
c) All three
d) None

Solution: b)

Statement 3 is incorrect.

Biogas contains about 55-65 per cent methane, 35-44 per cent of carbon dioxide and traces of other gases such as
hydrogen sulphide, ammonia and nitrogen.
In its raw form, biogas can be used as a clean cooking fuel like LPG for lighting, motive power and electricity
generation.
Further, biogas can be purified and upgraded up to 98 per cent of purity to make it suitable to be used as a
green fuel for transportation or filling of cylinders. The process relies on a high pressure of circa 250 bar and
hence is called compressed biogas (CBG).

The Sustainable Alternatives Towards Affordable Transporation (SATAT) scheme by the Ministry of Petroleum
and Natural Gas (MoPNG) encourages entrepreneurs to set up CBG plants, produce and supply CBG to oil and
gas marketing companies (OGC/OMC) for selling it as automotive and industrial fuels.

www.insightsonindia.com 52
INSTA STATIC QUIZ
35) Consider the following statements.
1. Soil Organic Matter (SOM) is mainly composed of carbon, hydrogen and oxygen but also have small
amounts of nutrients such as nitrogen and phosphorous.
2. Storing the carbon contained in organic matter within the soil, is seen as one way to mitigate climate
change.
3. Soil Organic Carbon (SOC) cannot be emitted back in gaseous form or lost into the atmosphere.
How many of the above statements is/are correct?
a) Only one
b) Only two
c) All three
d) None

Solution: b)

Statement 3 is incorrect.

Soil Organic Matter (SOM) is mainly composed of carbon, hydrogen and oxygen but also has small amounts of
nutrients such as nitrogen, phosphorous, sulphur, potassium, calcium and magnesium contained within organic
residues.

After carbon enters the soil in the form of organic material from soil fauna and flora, it can persist in the soil for
decades, centuries or even millennia. Eventually, SOC can be lost as CO2 or CH4 emitted back into the
atmosphere, eroded soil material, or dissolved organic carbon washed into rivers and oceans.

36) Consider the following statements.


1. Global warming potential (GWP) is a measure of how much energy the emissions of 1 ton of any
greenhouse gas (GHG) will absorb over a given period of time, relative to the emissions of 1 ton of carbon dioxide
(CO2).
2. The larger the GWP, the more that a given GHG warms the Earth compared to CO2 over that time
period.
3. The time period usually used for GWPs is 10 years.
How many of the above statements is/are correct?
a) Only one
b) Only two
c) All three
d) None

Solution: b)

Statement 3 is incorrect.

The Global Warming Potential (GWP) was developed to allow comparisons of the global warming impacts of
different greenhouse gases. Specifically, it is a measure of how much energy the emissions of 1 ton of any
greenhouse gas will absorb over a given period of time, relative to the emissions of 1 ton of carbon dioxide (CO2).
The larger the GWP, the more that a given gas warms the Earth compared to CO2 over that time period. The time
period usually used for GWPs is 100 years. GWPs provide a common unit of measure, which allows analysts to
add up emissions estimates of different gases (e.g., to compile a national GHG inventory), and allows
policymakers to compare emissions reduction opportunities across sectors and gases.

CO2, by definition, has a GWP of 1 regardless of the time period used, because it is the gas being used as the
reference.

37) What would happen if phytoplankton of an ocean is completely destroyed for some reason?

1. The ocean as a carbon sink would be adversely affected.


www.insightsonindia.com 53
INSTA STATIC QUIZ
2. The density of ocean water would drastically increase.
3. The food chains in the ocean would be adversely affected.
How many of the above statements is/are correct?
a) Only one
b) Only two
c) All three
d) None

Solution: b)

Statement 2 is incorrect.

Phytoplanktons are food factories of the ocean, it fixes atmospheric carbon dioxide in large quantities (Carbon
Sink) and produce food. So, if phytoplanktons are removed completely it leads to catastrophic failure of ocean
ecosystem. It doesn’t influence the density of ocean water.

38) Which of the following are the Ozone depleting substances.

1. Hydrobromoflurocarbons (HBFCs)
2. Halons
3. Methyl bromide
4. Carbon tetrachloride
5. Methyl chloroform
Select the correct answer code:
a) All except 3
b) All except 4
c) All except 5
d) All of the above

Solution: d)

Ozone depleting substances include:


chlorofluorocarbons (CFCs)
hydrochlorofluorocarbons (HCFCs)
hydrobromoflurocarbons (HBFCs)
halons
methyl bromide
carbon tetrachloride
methyl chloroform.

39) Consider the following statements regarding ‘Zombie fires’ recently seen in news

1. A zombie fire is a fire that continues to burn underground and then reignites on the surface after a
period of time.
2. It burrow deep in organic soils such as peat lands.
3. There are evidences of zombie fires scattered across the Arctic region.
How many of the above statements is/are correct?
a) Only one
b) Only two
c) All three
d) None

Solution: c)

www.insightsonindia.com 54
INSTA STATIC QUIZ
Dormant "zombie fires" are scattered across the Arctic region. A zombie fire is a fire that continues to burn
underground and then reignites on the surface after a period of time. Embers deep in organic soils such as peat
lands can spark into flames weeks, months and even years later.

40) Consider the following statements.

1. Biochar induces carbon sequestration.


2. Biochar is used as a soil amendment.
3. Use of Biochar in soil leads to soil-borne diseases.
Select the correct answer code:
a) 1, 2
b) 2 only
c) 1, 3
d) 1, 2, 3

Solution: a)

Biochar is found in soils around the world as a result of vegetation fires and historic soil management practices.
Intensive study of biochar-rich dark earths in the Amazon (terra preta), has led to a wider appreciation of
biochar’s unique properties as a soil enhancer.

Statement 1: Biochar is charcoal used as a soil amendment. Like most charcoal, biochar is made from biomass via
pyrolysis. Biochar is under investigation as an approach to carbon sequestration to produce negative carbon
dioxide emission.

Statement 2 and 3: Independently, biochar can increase soil fertility of acidic soils (low pH soils), increase
agricultural productivity, and provide protection against some foliar and soil-borne diseases.
Biochar reduces pressure on forests. Biochar is a stable solid, rich in carbon, and can endure in soil for thousands
of years

41) Which of the following ecosystem has an upright pyramid?


1. The pyramid of number in a tree ecosystem.
2. The pyramid of energy in a grassland ecosystem.
3. The pyramid of biomass in sea.
How many of the above statements is/are correct?
a) Only one
b) Only two
c) All three
d) None

Solution: a)

Only Statement 2 is correct.

• In most ecosystems, all the pyramids, of number, of energy and biomass are upright, i.e., producers are more
in number and biomass than the herbivores, and herbivores are more in number and biomass than the
carnivores. Also, energy at a lower trophic level is always more than at a higher level.
• There are exceptions to this generalization: If you were to count the number of insects feeding on a big tree
what kind of pyramid would you get? Now add an estimate of the number of small birds depending on the
insects, as also the number of larger birds eating the smaller. Draw the shape you would get.
• The pyramid of biomass in sea is generally inverted because the biomass of fishes far exceeds that of
phytoplankton.
• Pyramid of energy is always upright, can never be inverted, because when energy flows from a particular
trophic level to the next trophic level, some energy is always lost as heat at each step. Each bar in the energy

www.insightsonindia.com 55
INSTA STATIC QUIZ
pyramid indicates the amount of energy present at each trophic level in a given time or annually per unit
area.
• The number of trophic levels in the grazing food chain is restricted as the transfer of energy follows 10 per
cent law – only 10 per cent of the energy is transferred to each trophic level from the lower trophic level.

42) Consider the following statements regarding Ecological Niche.


1. A niche can be described as the interactions between a species with its biotic and abiotic environment.
2. Two species can occupy the same niche in the same environment for a long time.
Which of the above statements is/are correct?
a) 1 only
b) 2 only
c) Both 1 and 2
d) Neither 1 nor 2

Solution: a)

Niche is basically the ecological or functional role of a species in an ecosystem, especially with regards to food
consumption. A niche can also be described as the interactions, a species has with its biotic and abiotic
environment.
According to the competitive exclusion principle, no two species can occupy the same niche in the same
environment for a long time.

43) Consider the following statements regarding dissolved oxygen in an aquatic ecosystem
1. Its concentration in fresh water is usually more than the concentration of oxygen in air.
2. It increases with increase in temperature of a water-body.
3. Snow cover of ice on water reduces dissolved oxygen concentration.
How many of the above statements is/are correct?
a) Only one
b) Only two
c) All three
d) None

Solution: a)

Only statement 3 is correct.

Dissolved oxygen (DO) is a measure of how much oxygen is dissolved in the water - the amount of oxygen
available to living aquatic organisms. The amount of dissolved oxygen in a stream or lake can tell us a lot about its
water quality.

Oxygen concentrations are much higher in air, which is about 21% oxygen, than in water, which is a tiny fraction
of 1 percent oxygen.

Dissolved oxygen levels decrease with increasing temperature and atmospheric oxygen content is far higher
than dissolved oxygen levels.

Ice and snow reduce the amount of sunlight reaching aquatic plants, thereby reducing photosynthesis and
oxygen production.

44) Consider the following statements regarding Sloth Bear.


1. Sloth bears are endemic to the Indian sub-continent and 90% of the species population is found in
India.
2. The species has the same level of protection as tigers, rhinos and elephants in India.

www.insightsonindia.com 56
INSTA STATIC QUIZ
3. Wildlife SOS India, is an organisation involved in sloth bear conservation and protection for over two
decades.
How many of the above statements is/are correct?
a) Only one
b) Only two
c) All three
d) None

Solution: c)

Classified as `Vulnerable’ on the IUCN Red List, sloth bears are endemic to the Indian sub-continent and 90% of
the species population is found in India.

Wildlife SOS India, an organisation involved in sloth bear conservation and protection for over two decades.

Sloth bears are identified by their very distinct long, shaggy dark brown or black fur, distinct white V -shaped chest
patch and four-inch long ivory-coloured curved claws used for digging out termites and ants from rock-hard
mounds. Listed under Schedule I of The (Wildlife Protection) Act of India, 1972 the species has the same level of
protection as tigers, rhinos and elephants.

45) Consider the following statements.


1. The World Wildlife Day (WWD) is celebrated to mark the establishment of CITES.
2. CITES is an international agreement between governments, that aim to ensure that international trade
in specimens of wild animals and plants does not threaten the survival of the species.
3. The CITES Secretariat is administered by International Union for Conservation of Nature (IUCN).
How many of the above statements is/are correct?
a) Only one
b) Only two
c) All three
d) None

Solution: b)

Statement 3 is incorrect.

In 2013, the United Nations General Assembly (UNGA) proclaimed March 3 as the UN World Wildlife Day to
celebrate and raise awareness of protecting the world’s wild animals and plants. This was as the Convention on
International Trade in Endangered Species of Wild Fauna and Flora (CITES) was signed in 1973 on this day.

CITES is an international agreement between governments. Its aim is to ensure that international trade in
specimens of wild animals and plants does not threaten the survival of the species.

www.insightsonindia.com 57
INSTA STATIC QUIZ
It accords varying degrees of protection to more than 37,000 species of animals and plants, ranging from live
animals and plants to wildlife products derived from them, including food products, exotic leather goods,
medicines, etc.

The CITES Secretariat is administered by UNEP (The United Nations Environment Programme) and is located in
Geneva, Switzerland. The Conference of the Parties to CITES is the supreme consensus -based decision-making
body of the Convention and comprises all its parties.

46) Why do exotic species post a threat to an indigenous ecosystem such as in a Lake or an isolated island in the
Andamans?
1. Such species may cause diseases in native species.
2. They may be predators of local species.
3. Such species compete with the local or native species for food.
How many of the above statements is/are correct?
a) Only one
b) Only two
c) All three
d) None

Solution: c)

Exotic species introduced to new environments often reset the ecological conditions in that new habitat,
threatening the species that exist there; this is the reason that they are also termed invasive species.
Invasive species that are closely related to rare native species have the potential to hybridize with the native
species; harmful effects of hybridization have led to a decline and even extinction of native species.
Invasive species can change the food web in an ecosystem by destroying or replacing native food sources. The
invasive species may provide little to no food value for wildlife.
Lakes and islands are particularly vulnerable to extinction threats from introduced species.

47) Consider the following statements.


1. The biomass in the upper trophic levels is generally very high as compared to the lower trophic levels.
2. The energy pyramid of an ecosystem is always upright and narrows to the top.
Which of the above statements is/are incorrect?
a) 1 only
b) 2 only
c) Both 1 and 2
d) Neither 1 nor 2

Solution: a)

Since each higher trophic level receives only a fraction of energy of the lower trophic levels, the energy pyramid
is narrow at the top.
But, generally (barring some aquatic ecosystems) lower trophic levels have higher biomass as compared to the
higher trophic levels.

www.insightsonindia.com 58
INSTA STATIC QUIZ

48) Estuaries are among the most productive ecosystems in the world. What could be the possible reason?
1. Some estuaries forms an ecotone of freshwater and salty seawater.
2. Estuaries are not subject to disturbing forces like tides and waves.
Which of the above statements is/are correct?
a) 1 only
b) 2 only
c) Both 1 and 2
d) Neither 1 nor 2

Solution: a)

• Estuaries and their surrounding wetlands are bodies of water usually found where rivers meet the sea.
• Estuaries are home to unique plant and animal communities that have adapted to brackish water—a mixture
of fresh water draining from the land and salty seawater.
• An ecotone is a transition area between two biomes. It is where two communities meet and integrate.
• It has some of the characteristics of each bordering biological community and often contains species not
found in the overlapping communities.
• For e.g. the mangrove forests represent an ecotone between marine and terrestrial ecosystem. Other
examples are grassland (between forest and desert.)

Estuaries are subject both to marine influences—such as tides, waves, and the influx of saline water—and to
riverine influences—such as flows of fresh water and sediment.
The inflows of both sea water and fresh water provide high levels of nutrients both in the water column and in
sediment, making estuaries among the most productive natural habitats in the world.
The banks of many estuaries are amongst the most heavily populated areas of the world

49) Which of the following statements is incorrectly stated?


a) Alpha diversity refers to diversity within a particular area or ecosystem.
b) Food chain is more comprehensive than food web in displaying possible transfers of energy.
c) Gamma diversity is studied at a very large scale, say a biome
d) None of the statements (a), (b), and (c) are incorrect.

Solution: b)

Alpha diversity
Alpha diversity describes the species diversity within a community at a small scale or local scale, generally the size
of one ecosystem. When we casually speak of diversity in an area, more often than not it refers to alpha diversity.

Beta diversity
Beta diversity describes the species diversity between two communities or ecosystems. It is at a larger scale, and
looks to compare the species diversity between two separate entities that are often divided by a clear
geographical barrier like a river or a mountain ridge.
www.insightsonindia.com 59
INSTA STATIC QUIZ

Gamma diversity
Gamma diversity is studied at a very large scale—a biome—where species diversity is compared between many
ecosystems. It could range over areas like the entire slope of a mountain, or the entire littoral zone of a sea shore.

Example: Let’s take a mountain slope as our landscape. On this slope, there will be many different patches of
forests and grasslands. Alpha diversity is the species diversity present within each forest or grassland patch of the
slope. Beta diversity is represented by the species diversity between any two patches and their communities.
Gamma diversity of the landscape is the species diversity along the entire range of the mountain slope.

Food web is more comprehensive than food chain.

50) Light pollution can have which of the following effects on the ecology and biodiversity?
1. It is known to disturb the reproductive cycles of some animals.
2. It adversely affects the migration of birds that navigate using the stars.
3. It disturbs circadian rhythms in humans affecting the sleep pattern.
How many of the above statements is/are correct?
a) Only one
b) Only two
c) All three
d) None

Solution: c)

Light pollution is excessive brightness that causes visual discomfort.


• Skyglow is the brightening of the night sky over inhabited areas. It is caused by cluttered, bright, excessive and
confusing groupings of light sources.
• It affects the reproductive cycles of those animals that depend on sensing light movements or seasonal
movements to reproduce. Since birds migrate using star light, it can confuse them, and even disorient night -flying
insects.
• Excessive blue light emitted from LEDs directly affects sleep pattern in Human by suppressing the production of
the hormone melatonin, which mediates the sleep-wake cycle in humans.

www.insightsonindia.com 60

You might also like